NAVLE 1 Flashcards Preview

NAVLE > NAVLE 1 > Flashcards

Flashcards in NAVLE 1 Deck (267)
Loading flashcards...
1
Q

A mixed-breed dog weighing 10 kg is being evaluated after having been trapped in a window well without access to food or water for an unknown period of time. The dog’s mucous membranes feel dry to the touch, and a moderate degree of decreased skin turgor is present. The dog’s HR is 220 bpm. The dog is responsive and is standing and walking normally, and pulse quality is normal.

If the degree of dehydration in this dog is approximately 7%, then how many milliliters of isotonic fluid should be administered to replace the dehydration deficit?

A 100 mL

B 250 mL

C 500 mL

D 700 mL

E 1500 mL

A

Yes, the correct answer is D) 700ml

Fluid needed -
BW(kg) x %dehydration = amt. given (litres) (Ref:Pasquini & Pasquini)

So for a 10kg dog with 7% dehydration

10 x 0.07 =0.70L = 700 ml

or if the weight is given in pounds :)

(BW(lb) x %dehydration) / (2.2) = amt. given (litres)

2
Q

A SLAP heartworm antigen test with a reported sensitivity of 96% and specificity of 98% is being used. Assuming the prevalence of heartworm in the area is 10%, what is the predictive value positive (PVP) of the test?

A 80%

B 92%

C 84%

D 99%

E 88%

A

Pick an imaginary number of animals that you test and fill out 2 x 2 table

If prevalence is 10% there must be 100/1000 dogs with heartworm and 900 dogs that are disease free.

A 96% sensitive test will correctly call 96/100 positive (box a) and in correctly call 4/100 negative (box c)

If 100/1000 animals are infected, than 900/1000 are disease-free. Your 98% specific test will correctly call 882/900 disease-free (box d: 0.98 x 900= 882) and in-correctly call 18/900 positive (box b– false positives)

a b c d boxes are filled it… calculate

PVP = a/(a+b)= 96 (96 + 18) = 84%

(pick 1000)

3
Q

Which of the following causes the highest pre-weaning mortality of piglets in swine operations?

A Crushing by the sow

B Porcine circovirus- associated disease (PCVAD)

C Porcine reproductive and respiratory syndrome (PRRS)

D Scours

E Transmissible gastroenteritis (TGE)

A

The best answer is A

4
Q

How/ why does lead cause the following cytological features?

* Basophilic stippling, poikilocytosis, anisocytosis, polychromasia and metarubricytosis

A

* The main cause of lead’s toxicity is its interference with a variety of enzymes because it binds to sulfhydryl groups found on many enzymes. Lead can mimic many other metals (zinc, iron), important in biological processes, which act as cofactors in many enzymatic reactions, displacing these metals at the enzymes on which they act. But it won’t properly functon as the cofactor. Therefore it will interfere with the enzymes ability to catalyze its normal reaction or reactions.

5
Q

Treatment of ivermectin toxicity. Do you give atropine? Why or why not?

A

* Treatment for ivermectin toxicity consists of supportive care (that one case study with ILE, not on collies)

* Ivermectin’s effect on vertebrates is that of a GABA agonist, not an acetylcholinesterase inhibitor. IV lipid therapy can be helpful, as can treatment with activated charcoal.

* But organophosphate and carbamate insecticides are acetylcholinesterase inhibitors and in cases of intoxication with these agents, which cause muscarinic signs, atropine is used to counteract those “SLUD” effects– salivation, lacrimation, urination and defecation.

6
Q

If you treat a down cow for milk fever in the morning, is it okay for your assistant to return to the farm to give Calcium again?

A

No, because the cow may have a more serious problem e.g. septic mastitis, twin left inside

7
Q
A

Bony lesion in the navicular bone that has a shape similar to a lollipop.

8
Q

Three pigs in a group of 30 weaner pigs died yesterday. Four additional pigs were found dead this morning. A few pigs in the pen are down and paddling. Some pigs are blind and other appear uncoordinated. These weaners have been off- feed for 3 days. Cultures from necropsy of the dead pigs grow:

* Lungs: Hemophilus parasuis, Streptococcus suis, Pasteurella multocida;
* Pharynx: Hemophilus parasuis, Streptococcus suis, Pasteurella multocida;
* Brain: Streptococcus suis;
* Heart: Streptococcus suis; and
* Pleura: Mixed growth, including Proteus spp.

=============================================================
Bonus Question 01

Which of the following is the most likely cause of illness in these pigs?

(A) Actinobacillus suis
(B) Hemophilus parasuis
(C) Pasteurella multocida
(D) Proteus spp
(E) Streptococcus suis

A

The answer is Streptococcus suis because isolation of S. suis from the brain was most likely to assist in the definitive diagnosis of disease.

9
Q

Which of the following is most likely true of clinically healthy pigs in a typical confinement swine operation?

(A) The pigs do not carry Streptococcus suis.
(B) The pigs are carriers of multiple serotypes of Streptococcus suis.
(C) The pigs are carriers of Streptococcus suis serotype 2 only.
(D) The pigs are carriers of Streptococcus suis serotype 3 only.
(E) The pigs are carriers of Streptococcus suis serotype 9 only.

A

B. The pigs are carriers of multiple serotypes of S. suis

10
Q

What are three testing options for newborn foals when neonatal isoerythrolysis (NI) is suspected? Which one is the easy one that can be done quickly on the farm?

A
  1. The stall-side, easy test is the jaundice foal agglutination test (mare’s colostrum + foal’s blood). It has a good correlation to actual hemolysis
  2. Another option is to perform a hemolytic test (mare’s serum + foal’s RBC + complement). Not easily done in the barn, so not as realistic a choice
  3. Whole blood cross- match– expensive and would take time

** side note there are more than 32 blood group antigens in horses (8 blood types, each with various allo-antigens). Types Aa and Qa are the most often associated with NI reactions

11
Q

Advantages and disadvantages of collecting peritoneal fluid via abdominocentesis by a needle with syringe vs. use of a blunt teat cannula.

A

I would suggest using an 18G 1.5-inch or even 3-inch needle (spinal needle). Horse abdominal walls are thick! Also, when using a needle for this purpose, if you pick a good spot and get fluid, you often do not need to attach a syringe. Simple redirection of the needle tip or a twist of the needle can get fluid to flow.

Needles

1) Fewer preparation steps when using a needle vs cannula.
2) Fairly cheap and easy to have a ready supply.
3) Especially in horses with large colon impactions, I think one is more likely to enter a viscus (do an enterocentesis by mistake).
4) If enterocentesis occurs, the local contamination is small with a needle vs with a cannula.

Teat cannulas

1) Requires more preparation (stab incision); maybe more “practice” is required to get the technique right.
2) VERY easy to reposition the teat cannula without traumatizing tissues. This is why I love them.
3) Requires sterilization after each use.

12
Q

Are Clostridia gram positive or gram negative? What is associated with Tyzzer’s disease?

A

* Gram variable- “Clostridium are anaerobic or microaerophilic spore-forming rods that do not produce spores in the presence of air, are usually gram positive….although Tyzzer’s bacillus has often been reported to be gram negative in tissue sections, it can appear gram variable and gram positive. Many clostridia stain gram variable or even gram negative once they have aged.”

* Clostridium piliforme- Tyzzer’s disease in horses

13
Q

What is Tyzzer’s Disease?

A

“Clostridium piliforme [is] a motile, sporeforming, rod shaped, flagellated, obligate, intracellular bacterium….

”The pathogenesis is poorly understood.

”Infection most likely results from oral exposure, eg, the ingestion of spores shed in the feces of infected animals. Possible sources include infective spores from the environment, contact with carrier animals, and in neonatal foals, ingestion of feces from the dam.

The primary site of infection is the lower intestinal tract with subsequent dissemination via the blood or lymphatics. The bacterium has an affinity for the intestine (epithelial and smooth muscle cells), hepatocytes, and cardiac myocytes. Stress factors such as capture, overcrowding, shipping, and poor sanitation appear to be predisposing.”

And a note for you exotics folks: “Sulfonamide administration predisposes rabbits to the disease.”

14
Q

Visual signs of dorsal displacement of the soft palate? Epiglottal entrapment?

A

* DDSP- if the epiglottis cannot be seen, it is completely covered by the dorsally displaced soft palate

* Epiglottic entrapment- when this occurs, the epiglottis itself is entrapped by the aryepiglottic fold. This fold envelops the apex of the epiglottis, but you can still see the shape of the epiglottis. It also sits on top of the soft palate (unlike DDSP)

15
Q

What is Equine Infectious Anemia (EIA)

A

EIA noncontagious– but transmitted by blood sucking insects– Retroviridae

* May test for often but will rarely see because of rigorous testing.

Presentation: EIA

  • Intermittent fever, weight loss, icterus, can cause abortions. Disease may be inapparent until routine EIA test.
  • Typically see in Mississippi River valley and in SE states (Texas to Florida; Texas is a hot spot).

Test of Choice:

  • Coggins AGID (1- to 2-day wait for results from state lab)
  • Newer ELISA tests allow horse-side Dx in minutes, but one still must confirm a Dx with Coggins.

Rx:

  • NONE: typically euthanize positives or isolate for life
  • No vaccine, no Rx can clear carrier state.

Prevention/Actions:

  • REPORTABLE.
  • Stable fly control; isolate foals of positive mares until disease free of EIA (takes up to 6 months, until can be sure maternal ABs are gone).
16
Q

When should you use AMs for strangles (streptococcus equi subsp equi)?

A

* Majority of vets will postpone therapy until after abscesses have begun to drain

“The Merck Veterinary Manual” has this to say: “Antimicrobial therapy is controversial. Most authors agree that initiation of antibiotic therapy after abscess formation may provide temporary clinical improvement in fever and depression, but ultimately PROLONGS the course of disease by delaying maturation of abscesses. Antibiotic therapy IS indicated in cases with dyspnea, dysphagia, prolonged high fever, and severe lethargy/anorexia.” (Note: The caps for emphasis are mine.)

17
Q

How to wash a mare’s RBCs for blood transfusion?

A

“The reference is Smith’s “Large Animal Internal Medicine” textbook (3rd edition, 2002). The author advocates use of a large volume centrifuge, but serial sedimentation can be used effectively.

1) For a typical 500-kg mare, 6 - 8 L of whole blood can be collected from the dam. 3 - 4 L will provide enough RBCs for most cases.
2) Blood should be anti-coagulated with:

a. Acid-citrate-dextrose (ACD) OR
b. Sodium citrate (3.8% NaCitrate solution; 1 part NaCitrate/9 parts blood).

3) Let anti-coagulated blood settle for 1 - 2 hours OR centrifuge.
4) Draw off plasma aseptically. Add a similar or greater volume of sterile isotonic saline (0.9% NaCl) and mix gently. Let settle for 1 - 2 hours or centrifuge.
5) Draw off saline and discard.
6) One can repeat washing OR re-suspend RBCs in equal volume of 0.9% NaCl and administer.

NOTE: One should follow basic guidelines for administering a blood product, i.e., monitoring body temperature, heart rate, respiratory rate, and flow rate of transfusion. Start slowly!”

18
Q

Causes of equine abortion

A

Non-infectious causes

* evil twin (number one cause of non-infectious abortion)

* Evil caterpillars- Mare Reproductive Loss Syndrome (MRLS)

* Evil fungus- fescue toxicosis infected by endophyte Acremonium

Infectious equine abortion

* Equine rhinopneumonitis (Equine Herpesvirus 1)

* Equine viral arteritis

* Bacterial abortion (Potomac horse fever, Leptospirosis, Streptococcus zooepidemicus, misc.)

* Equine mycotic placentitis

19
Q

What is Verminous Myelitis? What is Equine Protozoal Myelopathy (EPM)?

A

* Verminous Myelitis- neurological disease due to aberrant helminth migration. Signs vary, but they are usually acute and asymmetric and may be progressive.

* EPM- neurological disease due to protozoa- Sarcoystis neurona, is the principal rule-out for multifocal, asymmetric, neurological disease in horses. It can mimic any neurologic disease, it just depends where the protozoa have caused inflammation in the nervous system, but it is typically characterized by gait abnormalities, muscle atrophy, and down horses.

20
Q

What is Cervical Stenotic Myelopathy? (CSM)

A

Equine Wobbler Syndrome– caused by stenosis of cervical vertebral canal. Look for younger horses (1-3 years) with normal mentation, but pronounced hindlimb ataxia, a clumsy “tin soldier” gait, knuckling, and stumbling. You can make the hind end sway when pulling on the tail (positive tail test, suggesting paresis). Exacerbation of symmetric signs with elevation of the head, lack of cranial nerve signs, atrophy, or systemic illness in a young horse suggest CSM.

21
Q

What is Equine Occipitoatlantoaxial malformation (OAAM)?

A

Due to a congenital C1- occipital malformation, particularly in Arabian horses. In dogs, atlantoaxial subluxation is most commonly a congenital problem of young toy or miniature breeds.

22
Q

What is equin degenerative myelopathy?

A

EDM- also called encephalomyelopathy

A common cause of symmetrical ataxia of all 4 limbs in younger horses, 6-8 months up to more than a year old.

23
Q

What are the four types of hypersensitivity reactions?

A

Antibody-mediated mechanisms govern types I, II, and III and occur rapidly while type IV reactions are cell-mediated and take longer. Most diseases actually involve a combination of types of immune-mediated reaction, but this basic classification system is still helpful in understanding allergic disease.

Type I reactions involve IgE and can be generalized (such as anaphylactic shock) or may be more focal reactions, such as urticaria or hives. Type I reactions include facial edema, salivation, vomiting, dyspnea, diarrhea, shock, collapse, and death. In the canine (because the liver is most affected) portal hypertension and visceral blood pooling cause the resultant GI signs.

In type II reactions cell-surface antigens are bound to antibody, forming an antibody-antigen complex that is capable of activating complement and causing cell lysis or causing an antibody-mediated cytotoxicity. IMHA and transfusion reactions involve cytotoxicity. Antibodies bound to cell receptors may also block the activation of the cell (e.g., anti-receptor antibodies in myasthenia gravis).

Type III hypersensitivity is immune complex-mediated. Endothelial deposition of immune complexes causes an inflammatory response and vascular damage. Vasculitis (usually in joints, skin, kidneys, lungs, or brain) results. The site of endothelial deposition of the immune complexes determines resulting signs. Glomerulonephritis, systemic lupus erythematosus, and hypersensitivity pneumonitis in cattle are type III hypersensitivities (as is “farmer’s lung” in people).

Immune complex-mediated hypersensitivity may result from neoplasia or chronic infections, but sometimes the cause of the disease remains obscure.

Type IV hypersensitivity is a delayed type of hypersensitivity. This results when sensitized lymphocytes (TH cells) respond to foreign antigen (or to molecules bound to cells). Contact hypersensitivity (as when some dogs react to plastic collars or bowls) and keratitis sicca are common examples of this type of reaction.

24
Q

Diagnosing pneumonia in horses

A

* Radiographs can be used as part of the work up but are not usually the first step nor done alone because you are limited inwhat you can image in an adult horse because of the size and thickness of the chest

* With a bacterial pneumonia, you may be able to show a cranioventral opacity, abscesses (multiple opacities), or a straight line (pleural fluid). You would want to combine with other diagnostics (auscultation/physical exam, ultrasound/echo, [esp. with pleural fluid] TTW, CBC, thoracocentesis, endoscopy, etc.).

Foals are easier to radiograph, being smaller. In a foal with a Rhodococcus equi infection, for example, you might be able to see a “cotton ball” appearance to the lungs, which is where there are abscesses.

25
Q
A

Acute bronchointerstitial pneumonia, foal, ultrasound

* Thoracic ultrasound examination of a 5 month old TB foal with acute bronchointerstitial pneumonia. Note the comet tails radiating from the visceral pleural surface

26
Q
A

Radiograph of acute bronchointerstitial pneumonia, foal

* diffuse interstitial pattern in the caudodorsal lung fields with marked peribronchial infiltration

27
Q
A

Pleuropneumonia, horse

* Thoracic ultrasonographic exam of a 2 yo TB with pleuropneumonia, demonstrating pleural effusion, pulmonary atelectesis, and fibrin accumulation on the visceral pleura

28
Q
A

Rhodococcal pneumonia, foal

* Thoracic ultrasonographic examination of a 3 month old foal with rhodoccoal pneumonia. Note pulmonary consolidation with air-filled lung visible in the periphery of the image. Small, fluid-filled abscesses can be identified in the pulmonary parenchyma.

29
Q

Why should you use extreme care when deciding to administer tetanus antitoxin in a horse? When is okay to use?

Tetanus Anti-toxin vs. tetanus toxoid?

A

* Tetanus antitoxin carries a risk of Theiler’s disease. Use only in horses with would who are UN- immunized or those of questional immunization, or foals from mares not immunized in late gestation. ( The majority of equine practioners urge their clients with broodmares to vaccinate the mare in the last month or so of gestation with tetanus toxoid- repeat annually 1 month prior to foaling, if pregnant)

* Anti-toxin- used to prevent/ treat a case of tetanus

* tetanus toxoid- used frequently to vaccinate and not associated with Theiler’s disease

30
Q

Main causes of equine lameness? Which blocks are used for each?

A

* Laminitis, Caudal heel pain syndrome (Navicular disease), Hoof abscesses/ punctures, equine osteochondrosis

  1. Heel block– aka palmar digital, used to help diagnose navicular disease
  2. Pastern block– aka foot block, abaxial sesamoid nerve block, you can use this to make a horse with acute laminitis comfortable
  3. Fetlock block (also called low palmar, volar, or low 4 point block)

** If a horse is still lame after these 3 blocks (heel, pastern, fetlock) continue up the limb with metacarpal/high 4-point, then proximal metacarpal blocks next

31
Q

Approximately 65% of 20,000 6-week-old broiler chickens in a poultry unit are ill. This poultry house is brand-new, and this is the first group of broilers to occupy it. Birds are huddled together and appear pale and droopy. Droppings are watery and blood-tinged. Feed consumption is decreased. About 200 birds died yesterday, and many are moribund this morning.

Which of the following is the most likely cause of the signs in these birds?

(A) Infectious bursal disease
(B) Colibacillosis
(C) Eimeria tenella
(D) Heterakis gallinarum
(E) Histomonas meleagridis

A

The correct answer is C, Eimeria tenella.

32
Q

How does one synchronize estrus in a herd? Which is the drug of choice to induce abortion during the first 5 months (does one use PGF) and after five months (does one use PGF plus dexamethasone)? How can one induce parturition (e.g., if one administers EITHER PGF or dexamethasone during the 9th month, would this do the trick)?

A

A: The standard way to synchronize estrus in heifers with prostaglandin F2a is to give one shot today and then return in about 2 weeks. Authorities vary about the exact interval. (Lutalyse “10 - 12 day interval” per Pasquini and “11 - 12 days apart” per “The Merck Veterinary Manual”; 2 shots, 14 days apart will synchronize estrus and ovulation in most cows.)

Just remember, with the PGF2a DUAL METHOD of estrus synchronization, the second shot comes about 2 WEEKS later.

To stop a pregnancy from occurring after a mismating (by the wrong bull, for example), use prostaglandin F2alpha to regress the corpus luteum. Resumption of the estrous cycle prevents fetal implantation. Dexamethasone will initiate abortion in late-term cows, but it is not indicated in very early pregnancies.

Typically, use dexamethasone (20 - 30 mg, IM), given within 2 weeks of normal term, to induce parturition in cattle. Retained placentas are more common after induced births compared to regular ones.

33
Q

What test(s) are typically chosen to diagnose Johne’s disease? I understand that there are a LOT of different types of tests available, to be selected based on suspicion of disease and particular disease stage, and that each has a problem with sensitivity, specificity, cost, availability, etc. So how do bovine practitioners diagnose Johne’s in an individual or herd? What’s an example of a practical, realistic work-up?

A

In general, think of ELISAs for herd testing.

If you see a cow with classic, clinical Johne’s (watery diarrhea, wasting away, but eats and acts fine), you don’t need a test to know you are probably looking at Johne’s. Get rid of that cow and worry about the herd.

Acid-fast staining of an impression smear made from the ileum of a cow with typical pathology can give a quick, low-cost (but insensitive), preliminary diagnosis. Biopsy of full-thickness sections of ileum and regional lymph nodes for culture and histopathology may provide a definitive diagnosis, but this is usually done only on particularly valuable animals.

A positive fecal culture is definitive, but it takes months to get a result, and the quality of results depends on the experience of the lab.

PCR on fecal material is costly. This can work well in the hands of some labs, but not all. PCR is usually not your first choice because of cost.

It is the subclinical shedders you are more worried about.

ELISA serologic testing used on a herd basis is the most cost-effective, timely way to screen herds.

34
Q

With only the clinical presentation as a guideline, how does one differentiate between tetanus and grass tetany?

A

The diseases resemble each other. Both are generally a presumptive Dx based on clinical signs and history. Both can present with convulsions and hyperexcitability in response to sound and tactile (touching) stimuli.

True tetanus, especially in later stages, is characterized by muscle rigidity (“lockjaw” [masseter], erect ears, pump-handle tail, or a rigid “sawhorse” stance). In horses, look for protruding nictitans. Generally it affects a single animal.

Bloat is a common sequelae in adult cows. A classic presentation of true tetanus is a calf that had an umbilical infection some time ago; and now you find her standing in a rigid, sawhorse stance. Rx is antitoxin (if early in the course of the disease, but this is costly), high-dose penicillin, muscle relaxants, and support (i.e., a quiet, dark stall; good footing; and nutritional support).

Grass tetany is one cause of hypomagnesemic tetany. Generally it’s more a disease of convulsions (but also anorexia, bellowing, isolation, alertness, frenzy, staggering, and recumbency). It may affect multiple animals; check blood or urine Mg in multiple animals to see if this is a herd problem. It’s most commonly a problem of lactating cattle (Mg+ is lost in the milk) or cattle on lush grass or green cereal crops or calves on an all-milk diet.

So, if your patient is a convulsing, tetanic feedlot steer, chances are he does NOT have hypomagnesemic tetany.

Rx is emergency Mg supplementation. In adults, we often use an IV commercial milk-fever Ca/Mg combo (Ca borogluconate + 5% Mg hypophosphate) ± Mg oxide orally or SQ. Relapse is common within 3 to 6 hours.

In calves, use IV 6% Mg borogluconate.

The prognosis is good if diagnosed early, but guarded if the animal is in convulsions and tetany.

35
Q

5 Common Drugs between Canada and the US prohibited in food-producing animals

A

The Canadian and U.S. lists BOTH contain these 5 drugs. For NAVLE®, I would know these 5 and call that good enough.

  • CHLORAMPHENICOL
  • CLENBUTEROL
  • DIETHYLSTILBESTEROL (DES)
  • NITROIMIDAZOLES
  • NITROFURANS
36
Q

Bovine ectoparasite treatment

A

There are lots of ways to treat ectoparasites.

Ear tags are plastic permeated with either organophosphate or pyrethrins. Sometimes the pyrethrins have been chemically modified in order to enhance efficacy. These are called synergized. They are put in the ears for the summer and changed at intervals. The insecticides are delivered locally and repel and kill face and horn flies and ticks on the face.

Pour-ons differ. Some are avermectins, which vary considerably in withdrawal times. Ivomec is absorbed extensively in tissues and has a 49-day withdrawal time for meat. It cannot be given to milk cows.

Moxidectin is an avermectin pour-on that does not have residual time in bovine tissues and does not appear in milk so it can be given to dairy and beef cattle without withdrawal of meat or milk.

Pyrethroid pour-ons are locally active. They are applied topically and adhere to skin and hair. They kill on contact for up to 30 days, and there is no systemic absorption so they don’t usually have a withdrawal time.

Insecticidal sprays are available for treatment of premises. These most often are pyrethroids, but in some states could be carbamate or organophosphates. OP and carbamate insecticides are banned in some states.

Back rubbers are large ropes that are anchored in 2 places. One end of the rope is dunked in a tank of diesel fuel and insecticide (see above sprays), and the cattle spontaneously rub their backs on the ropes and self-apply the insecticide.

Burlap bags of dry insecticide (see above sprays) with an inert carrier such as non-hydrated lime or talc can be mixed with the insecticide powder and placed at body level. Cows rub on the bags and release the powder from the bags, thus self-applying the insecticide.

37
Q

Milk Fever Treatment

A

Milk fever (postparturient paresis) is common, common, common in practice and an emergency. Other things can also make a recently fresh cow go down; so for a down cow who recently gave birth, it’s a good habit to check for septic mastitis and also check the uterus for a twin still inside before you give the cow IV calcium.

If the cow is down, then total plasma calcium is around 4 mg/dl. A down cow with milk fever will die if you don’t treat. So in practice you don’t calculate at all, you just treat.

Typically we would give a down cow with milk fever 500 cc IV of a 23% calcium solution IV (SLOWLY!). With a severe case many vets today also follow up with an oral calcium-containing gel. It’s not uncommon for a cow with a severe case to need another treatment in 8 - 12 hours.

There are many mixes of calcium fluids out there with different minerals or dextrose in them. Some vets or farmers may give some SQ or IP (intraperitoneal). Just make sure you don’t leave a dextrose-containing solution SQ, or it could abscess.

If the plasma calcium is higher (in the 6- to 7-mg/dl range), the cow may only appear weak and be showing some vague neuro signs or be staring off at Mars or be vomiting (signs of milk fever). In that case, oral calcium may be sufficient. Usually an alert farmer will have caught those cases, and you aren’t called.

38
Q

How can you differentiate between the two types of bloat in a cow?

A

To tell the difference between types of bloat, pass a stomach tube and look at the contents. If it looks like a green shaving cream, it is frothy bloat; the cow needs to be treated with Therabloat (poloxalene). If it is free gas, the bloat will resolve by passing the gas through the stomach tube. Treatment depends upon the cause.

Lisle George, DVM, PhD, DACVIM

There is some confusion about the different kinds of bloat. I’ll try to summarize in a way that makes practice sense (at least to me).

Bloat is one of the bovine emergencies that will kill a cow in no time if you don’t do something fast, but you can also save them and look like a hero. (Milk fever is another like this.)

Typically, you don’t know exactly why they bloated. But, if it’s on a feedlot, then probably it’s feedlot bloat; and, if on a dairy, then maybe lush pasture started the bloat, or the cow got into the grain bin, or you just don’t know. It doesn’t matter.

Here is what you do, regardless.

Get the cow into sternal recumbency.

The short answer is that you put a Frick speculum (a metal tube) into the cow’s mouth and pass a stomach tube through it and down the esophagus into the rumen to see if you can relieve the bloat that way. You may need to wiggle it around some to get the end of the tube above the food mat. If you are foolish enough to put your mouth on the tube to try and blow it clear, this will be the exact moment when a liter of green slime comes blowing out into your face. (I mean, ‘um, not that anyone we know here ever did that, but, ‘um, just sayin’….)

If gas blows off, it was a gas bloat. We used to give a gallon of mineral oil ± Carmilax (laxative). Often, to be safe, we would throw in a bottle of Therabloat too (poloxalene, a nonionic, surfactant, antifoaming agent), whether it was particularly indicated or not.

If tubing doesn't relieve the bloat, you may have a frothy bloat that requires an antifoaming agent, such as mineral oil, often mixed with a surfactant called dioctyl sodium sulfosuccinate (DSS or docusate sodium)(VIN Community) LongLink @ www.vin.com...
or poloxalene (for pasture bloat). "Pasquini's Guide to Bovine Clinics," p. 26, mentions that you could also add a box of Tide laundry detergent in there. I can't say I ever did that....

Give the mineral oil and Therabloat through the tube. You will probably need to “pop the bubble” also. Drive a trocar into the highest point of the left side (left paralumbar fossa). This is pretty exciting (especially for the cow), but it can save her life.

You stitch or glue the tube part of the trocar to the cow and leave it in. Start the cow on antibiotics (like Naxcel). You can pump poloxalene into the trocar or via the gastric tube. You may need to treat several times. A serious, nonresponding case might need a rumenotomy and removal of feed contents. If you are careful about the way you exteriorize the rumen before you cut, you can prevent or minimize the problem of rumen contents leaking into the abdominal cavity. If rumen contents get into the abdominal cavity, clean things as best you can. Make sure the cow is on antibiotics.

A final note: Sometimes you run into something, such as an apple, that blocks passage of a stomach tube. You either must reach in and pull it out (and hope this cow isn’t drooling because of rabies) or work both hands from below the obstruction and massage it up and out.

39
Q

Vagal indigestion vs. bloat in a cow

A

Bloat and vagal indigestion overlap some, so yes, it can be confusing.

Vagal indigestion is kind of a catch-all term (my opinion) a motility disturbance of the forestomachs, usually seen in individual animals, developing slowly over days. When bloat happens, its usually quick, and an emergency.

Vagal indigestions are often associated with or secondary to traumatic reticuloperitonitis (hardware disease) and adhesions/peritonitis somewhere among the forestomachs.

One KIND of vagal indigestion (so-called “type 1”) can cause a free gas bloat because the cow cannot eructate. Once a cow can’t burp, they bloat up pretty fast- but the underlying pathology was probably evolving for days/weeks.

There are other kinds of vagal indigestion - again, basically motility problems due to any number of causes- adhesions, too-dry feed and/or not enough water, abomasal impaction, omasal impaction, secondary to displaced abomasum or liver abscesses/adhesions…the list goes on. This is what I meant about vagal indigestion being a “catch-all” term.

In the world of bloat, frothy bloat is called “primary bloat” and it generally due to lush legume/high protein pasture or fine feed/low roughage diets.

“Secondary bloat” is free gas bloat, as I mentioned above, basically happens is a cow can’t eructate (many reasons-foreign body[like an apple], tumor, external pressure/obstructions, secondary to esophageal lesions, stenosis, grain overload pH issues, tetanus and on and on).

In practice, you seldom know right off what is causing a bloat, you just have to treat it right away or the cow can die.
If you are called to a farm to look at a bloated cow, you don’t mess around. Get there fast.

Can you pass an orogastric tube?
-No? It may be choke. Feel the throat for something obvious, like an apple.
-Yes?
After you pass the tube, does a bunch of gas spew out?
-If yes, its a gas bloat. Let the gas blow off. Then treat.
-If no gas? Move the tube around some- the end may be under liquid
-Still No gas? Might be frothy. Treat accordingly

While you are doing all this, you can get history from the farmer, not to mention vital signs etc which will help you home in on causes and treatments.

40
Q
A

A- Laryngeal FB

41
Q

A 9-year old neutered male cat is presented, he is an indoor/outdoor cat who has recently been losing weight. He doesn’t run around like he used to and wheezes now when he does move around.

His owner relates that he hasn’t had much of an appetite the last few weeks and is vomiting every day or so lately.

On physical exam it is noted that the cat is thin, with mild dehydration and somewhat dark mucous membranes but a normal capillary refill time. On auscultation inspiratory wheezes and somewhat irregular, muffled heart sounds are heard.

T=103.1 F (39.5 C)…..[N=100-103.1 F, N=37.8-39.5 C]
HR=124 bpm…………..[N=100-140]
RR=30 brpm…………..[N=16-40]

A lateral chest radiograph looks like the image below.

What is likely to have caused this illness?

A - Hit by a car
B - Hypertrophic cardiomyopathy
C - Pleural effusion
D - Lung lobe torsion
E - Feline mediastinal lymphosarcoma

A

A- Hit by a car

* Diaphragmatic hernia (see loops of bowel with gas density overlapping lung fields). Almost always secondary to trauma especially in pets hit by cars. May present acutely after accident (shock, pale, tachypnea, tachycardia, dyspnea) or may show up with NO Hx of trauma and nonspecific GI, respiratory signs. May hear gut sounds in chest, muffled heart sounds. Can be congenital (think Weimaraner, Cocker), asymptomatic and go undiagnosed until middle age.

* In cattal associated with traumatic reticuloperitonitis (Hardware disease)

42
Q
A

Pleural effusion (floating lungs)

43
Q

A 20 yo Burmese python is presented for unusual lethargy after its meal of a rat. Most likely diagnosis?

A - Foreign body
B - Egg peritonitis
C - Pneumonia
D - Normal radiographs
E - Dysecdysis

A

A- FB

Linear amorphous heterogenous opacity consistent with fiber is visible throughout the proximal GI tract. A complete small animal skeleton, most likely the rat, is visible immediately caudal to the fiber material.

44
Q

A 6 month old terrier mix is presented with 3 days of vomiting, diarrhoea, and lethargy. Based on the radiograph and ultrasonography image from the abdomen, which one of the following choices is the most likely diagnosis?A - Splenic torsion
B - Intussusception
C - Renal mass
D - Microhepatica
E - Generalized enteritis

A

B- jejunal intussusception. There are markedly dilated bowel loops in the caudal portion of the abdomen.

More normal sized loops are gas- filled in the cranial abdomen. The peritoneal detail is poor, likely because of his young age.

U/S diagnosed the intussusception. You can see the bowel within bowel, or concentric rings of the intestine in both the longitudinal and the cross- sectional image.

There is often mesenteric fat pulled into the intussusception, which appears hyperechoic on the transverse image. It is more rare to see jejunal intussusception. Most intussusceptions involve the ileum and colon. A more proximal obstruction results in focal bowel dilation such as in this case.

Distal obstruction often causes dilation of the entire proximal or oral small intestine.

45
Q

A 6-year old quarterhorse gelding is presented in September in North America with a three-day history of depression, poor appetite, fever and worsening gait. The owner says the horse seems weak on his hind legs, stumbles and sometimes presses his head against the wall of his stall.

The horse is ataxic and hypermetric in all four legs. Serum antibody titers to EEE, WEE, and VEE are low. A Western Blot test of CSF for antibodies to Sarcocystis neurona is negative.

Which one of the following diagnostic tests should be performed next?

A - CSF tap and test for Japanese encephalitis
B - MRI to rule out nigropallidal encephalomalacia
C - Serum IgM capture ELISA for West Nile Virus
D - Plasma antibody test for St. Louis encephalitis
E - Serum AGID for equine infectious anemia

A

C- Serum IgM capture ELISA for WNV

Serum IgM capture ELISA for West Nile Virus. Think of the equine encephalidities in a febrile horse, especially in the fall months. Because serum antibody tests for EEE, WEE and VEE are low, West Nile virus encephalitis is the big remaining rule out.

Look for variable and nonspecific signs like depression, low-grade fever and anorexia in combination with neurologic signs like head-pressing (image), ataxia (often hind-end weakness or paralysis) and visual impairment. IgM capture ELISA is the test of choice.

Remember that in most states and provinces, you must REPORT a horse with clinical signs of encephalomyelitis, even if the test results are not in yet.

With equine infectious anemia (EIA), think of recurrent fever, weight loss, dependent edema, petechial hemorrhages (image) and icterus.

Think of ataxia and atrophy with equine protozoal myeloencephalitis (EPM) due to Sarcocystis neurona.

46
Q

While conducting a routine physical on a 4 year old male intact bulldog, an irregular heart rhythm with a slow rate that is markedly slower on expiration is audible during auscultation.

T=102.1 F (38.9 C)..[N=99.5-102.5 F, N=37.2-39.2 C]
HR=60 bpm…………[N=60-120]
RR=24 brpm………..[N=15-34]

What should be done next?

A - Nothing
B - ECG
C - Echocardiogram
D - Chest radiograph, CBC, blood chemistry panel
E - Refer to for cardiology consult

A

Nothing. Extreme accentuation of sinus arrhythmia (bradycardia), markedly slower during expiration is a normal finding in brachycephalic breeds. No treatment needed if dog is not symptomatic.

SYMPTOMATIC animals would present with fainting, weakness. If respond to an atropine test, consider Med Rx with glycopyrrolate, propantheline, isoproterenol. If poor response, may need a pacemaker.

47
Q

During a routine immunization visit for a 2 year-old neutered male Newfoundland dog, a systolic ejection-type (crescendo-decrescendo) murmur is detected, audible loudest on the left side of the chest between the 2nd and 5th intercostal (IC) space and at the thoracic inlet lateral to the trachea.

Which condition is highest on a differential diagnosis list?

A - Pulmonic stenosis
B - Mitral dysplasia
C - Tricuspid dysplasia
D - Aortic stenosis
E - Patent ductus arteriosus (PDA)

A
Aortic stenosis (also called sub-aortic stenosis [SAS]), is a systolic, ejection-type (crescendo-decrescendo) heart murmur which may be heard most loudly on the left chest between the 2nd and 5th intercostal (IC) space or at the thoracic inlet (lateral to trachea).
Inherited in Newfoundlands. Predilection in many BIG BREEDS- German Shepherd, Golden Retriever, Boxer, Rottweiler.

Mitral dysplasia and other mitral valve problems are heard further back on left at 5th-6th IC. More common in CATS.

With pulmonic stenosis see RIGHT ventricular hypertrophy, because pulmonic valves blocks outflow from R ventricle (mostly dogs).

Tricuspid dysplasia is heard further back on RIGHT at 5th-6th IC. Uncommon.

Expect a continuous murmur with patent ductus arteriosus (PDA). Vast majority detected at first vaccination visit.

48
Q

Several piglets in a group weaned 10 days ago in the nursery facility of a large commercial swine operation were found dead. On evaluation, some weaners have swelling around the eyes and forehead.

Some are in lateral recumbency and dyspneic. Necropsy of the dead piglets reveals subcutaneous and submucosal edema.

The most likely causative organism is…

A - Brachyspira hyodysenteriae
B - Escherichia coli
C - Lawsonia intracellularis
D - Clostridium septicum
E - Streptococcus suis

A

E. coli causes the characteristic lesions of Edema disease in recently weaned piglets. Marked swelling of the periocular region, forehead and submandibular area follow infection. Piglets may die peracutely. Usually only a few piglets in a group are affected, but affected piglets perish rapidly (within 12 hours). Hemolytic E. coli that produce F18 pili and Shiga toxin 2e are implicated in edema disease.

To make a definitive diagnosis, E. coli must be first isolated and then characterized as an edema disease strain (that is, producing F18 pili and Shiga toxin 2e).The course is so rapid that treatment is ineffective. Antibiotics may be administered to unaffected pigs in the group.

Brachyspira hyodysenteriae causes Swine dysentery also called bloody scours.

Lawsonia intracellularis causes Porcine proliferative enteritis (diarrhea, often with fibronecrotic casts).

Clostridium septicum is the agent of Malignant edema in many animals. Infection occurs through contaminated wounds and turns affected muscle dark brown or black.

Streptococcus suis causes septicemia and meningitis in weaners and growing pigs.

49
Q

In September, two Quarter Horse mares horses are presented that are pastured in a group of 5 in Oklahoma. The horses eat pasture grass supplemented by alfalfa hay and a small amount of grain.

One mare was found peracutely dead this morning. The other is depressed, anorexic and colicy.

Physical exam reveals dark, congested mucous membranes with small ulcer-like erosions. The mare makes frequent attempts to urinate, yielding red urine (hematuria) with a urine specific gravity (USG) of 1.006. She makes repeated attempts to drink small amounts of water and keeps her muzzle submerged in the water trough.

T=102.7 F (39.3 C)..[N=99-101.3 F, N=37.2-38.5 C]
HR=40 bpm…………[N=28-40]
RR=20 brpm………..[N=10-14]

Necropsy of the other mare shows that the stomach and bladder linings are irritated and hemorrhagic.

What is the diagnosis?

A - Clostridium perfringens type A
B - Arsenic toxicity
C - Sorghum cystitis
D - Cantharidin toxicity
E - Enzootic hematuria

A

This is Cantharidin toxicity caused by blister beetles (Epicauta spp) which swarm in alfalfa hay during harvest.

Cantharidin is a potent irritant: see colic, renal disease, hematuria, peracute death. Follow this link to see a Merck image of Hemorrhagic gastritis. Follow this link to see a Merck image of Hemorrhagic cystitis.

Enzootic hematuria is a cow disease thought to be caused by Bracken fern toxicity. In horses, see thiaminase-related STAGGERS with bracken fern and with Horsetail (Equisetum spp.) on U.S. West Coast.

Sorghum cystitis/ataxia is characterized by cystitis, urinary incontinence (“dribbling” ), posterior incoordination.

50
Q

A 12-year old male neutered cat weighing 14 pounds is presented with a 2-month history of PU/PD, increased appetite, lameness, weight gain, exercise intolerance and dyspnea.

Physical exam shows a systolic heart murmur with a gallop rhythm, a lateral chest radiograph shows pulmonary effusion and a large heart.

T=102.0 F (38.9 C)..[N=100-103.1F, N=37.8-39.5 C]
HR=110 bpm…….[N=130-140]
RR=24 brpm……..[N=16-40]

A CBC shows

PCV=48 %………….[N=24-45%], WBC=14,850/µl..[N=3800-19,500/µl]
Neuts=88%…………[N=35-75%], Lymphs=4%..[N=20-55%]
Monos=7%………….[N=1-4%], Eos=1%..[N=2-12%]
Basos=rare…………[N=rare]

Blood chemistry reveals the following
NA=150 mEq/L…………….[N=151-161 mEq/L]…K=5.1 mEq/L..[N=3.5-5.1 mEq/L]
LDH=200 U/L……………..[N=35-225 U/L]……….ALT=108 U/L..[N=8.3-53 U/L]
Total protein=10.1 g/dl..[N=5.7-8.0 g/dl]……..Bilirubin (total)=0.3 mg/dl [N=0.0-0.2 mg/dl]
Alk Phos=200 U/L………[N=3-65 U/L]…………..Cholesterol=250 mg/dl..[N=95-130 mg/dl]
BUN…………47 mg/dL (2.6 mmol/L) …………[Normal: 10-30 mg/dl (0.55- 1.66 mmol/L)]
Creatinine= 3.4 mg/dL (300.56 ?mmol/L) ..[N=0.5-1.6 mg/dL (44.2- 141.44 ?mmol/L)]
Glucose…..350 mg/dl (19.4 mmol/l)………..[N=63-132 mg/dl(3.5-7.3 mmol/L)]

Urinalysis
U Sp. G= 1.018……..[N=1.020-1.040]
Glucose +++, WBC ++, RBCs +, protein +++

Which one of the following choices is the most likely diagnosis?

A - Hyperthyroidism complicated by renal disease
B - Acromegaly
C - Pancreatic exocrine insufficiency
D - Hyperadrenocorticism
E - Diabetes insipidus complicated by cardiomyopathy

A

B- Acromegaly

This complicated mix of diabetes mellitus , renal disease and heart failure/cardiomyopathy in an OLDER MALE cat suggests feline acromegaly.

First presenting sign may be PU/PD, polyphagia of diabetes. WEIGHT GAIN in an unregulated diabetic cat STRONGLY SUGGESTS acromegaly, (but they may LOSE weight at first).

Follow this link to see pulmonary edema and a large heart.

May see prognathism (long mandible), lameness (esp. cats), marked vertebral spondylosis, thickened skin, large head, wide interdental spaces and a Stress leukogram (High neuts, low lymphs/Eos)

51
Q

A 5-year old Holstein cow is presented with a 2-day history of being off feed and a precipitous drop in milk production.

T=103 F (39.4 C)..[N=101.5-103.5 F, N=37.2-38.5 C]
HR=132 bpm……..[N=55-80]
RR=36 brpm………[N=10-30]

The cow stands with abducted elbows, an arched back and is reluctant to move. Physical exam shows intermandibular edema and bilateral jugular distention.

A grunt is heard when pressure is applied to her xiphoid and there is a washing machine murmur (almost like splashing sounds) on both sides. There is little rumen activity.

What is the recommendation for the farmer?

A - Treat with high-dose penicillin/streptomycin
B - Check the feed for excess monensin/lasalocid
C - Test the herd for bovine leukosis
D - Check the feed for cottonseed meal (gossypol)
E - Cull this cow

A

E- Cull this cow

Bilateral dilated jugulars say “Severe heart problem”. A washing machine murmur and the painful stance, as well as respiratory grunting all point to hardware disease (Traumatic reticuloperitonitis).
Treatment is unrewarding- CULL. Prevent problem by making every cow swallow a small bar magnet to attract and hold nails, wire, sharp metal inside the reticulum.

COOL FACT: A compass can tell you if a cow has a magnet. Hold it near the brisket. If there is a magnet, compass needle will point to the cow, even if you move the compass.

Cardiac lymphosarcoma may present with heart failure signs, but less likely to have such an acute onset of agalactia, painful stance or classic “washing machine” murmur.

Monensin/Lasalocid are ionophore coccidiostats associated with cardiac failure- MOST TOXIC to HORSES

Cottonseed meal contains gossypol - see cardiac toxicity/dyspnea/ sudden death in calves; sterility/decreased conception in adults. are associated with cardiac toxicity.

52
Q

A flock from a broiler chicken operation is presented to investigate a disease outbreak characterized by caseous accumulations in the throat and weight loss.

A typical bird looks like the image below.

Two diseases are suspected: it is either candidiasis (“thrush”) or another, similar disease.

What is the other disease and how might the diagnosis be confirmed?

A - Trichomoniasis, microscopic smear exam
B - Fowl cholera, tracheal aspirate culture
C - Infectious coryza, AGID
D - Aspergillosis, fungal culture
E - Necrotic enteritis, fecal flotation

A

This is Trichomoniasis , caused by Trichomonas gallinae, and diagnosed by microscopic smear exam of the caseous oral exudates. Look for trichomonads. More a problem in PIGEONS, but can cause disease in chickens.

Remember Trichomonas foetus in cattle causes infertility (early embryonic death actually, 1st 2 months pregnancy). Infected bulls are mechanical carriers to cows.

Candidiasis is a fungal disease that can look SIMILAR to trichomonas in chickens. Follow this link to see a Merck image of Candidiasis.

Caused by Pasteurella multocida, Fowl Cholera causes sudden onset septicemia with VARIABLE signs. (Sudden death, anorexia, depression, mucoid beak discharge, ruffled feathers, diarrhea, increased RR.

Aspergillosis presents as respiratory disease. See fungi on microscopic smear, may see granulomatous lumps in lungs.

See sudden death with Necrotic enteritis, caused by Clostridium perfringens. Follow this link to see the so-called “Turkish towel” intestinal pseudomembrane of Necrotic enteritis.

53
Q

A four-month-old Jack Russell terrier puppy was playing outdoors. Later that evening the owner noticed that the puppy was wheezing and coughing.

Based on the radiographs, what would be the best step to take next?

A - Bronchoscopy
B - Barium swallow
C - Edrophonium challenge test
D - Trans-tracheal wash
E - Dental prophylaxis

A

Bronchoscopy is the next best step. On radiographs of the thorax, there is an alveolar pulmonary pattern in the left cranial, and right middle lung lobes, with patchy increased opacity in the remaining lobes.

The trachea is narrowed at the thoracic inlet, with an apparent linear intraluminal opacity. There is dilation of the pharynx with air, and there is gas within the esophagus and gastrointestinal tract.

The tracheal narrowing and intraluminal opacity may indicate edema, mucus, or foreign material in this region. There is secondary upper airway obstruction as indicated by the dilated pharynx and aerophagia.

The alveolar pattern is due to bronchopneumonia.

54
Q

A 9 year old German shepherd is presented with unchecked bleeding from a cut on the gums above the right canine tooth. The owner relates that the dog has lost weight and had an episode of collapse 3 days ago, but he recovered.

On physical exam, the gums are pale with petechiae and ecchymotic hemorrhages. There is tachycardia and a palpable cranial abdominal mass.

A coagulation profile shows the following:

Thrombocytes= 82,533 per microliter..[N=200,000-900,000]
Buccal mucosal bleeding time (BMBT), increased
Activated partial thromboplastin time (aPTT), increased
Prothrombin time (PT), increased
Thrombin time (TT), increased
Fibrin degradation products (FDPs), increased

What disorder of coagulation best fits this pattern?

A - Von Willebrand’s disease
B - Idiopathic thrombocytopenia
C - Disseminated intravascular coagulation
D - Anticoagulant rodenticide toxicity
E - Hepatic insufficiency

A

A lab pattern of low platelets, increased bleeding time and across the board increases in aPTT, PT, TT and FDP tests suggests disseminated intravascular coagulation (DIC). DIC is not a disease in its own right- it is a complex hemostatic defect characterized by enhanced coagulation and fibrinolysis, secondary to other diseases. Fibrinolysis and depletion of clotting factors leads to hemorrhage.

Many, many diseases, all of them bad, can precipitate DIC. This case presentation (pale, older German shepherd with Hx of collapse, bleeding and an abdominal mass) suggests hemangiosarcoma.

Remember your “H diseases” associated with DIC:
Heartworm
Heart failure
Hemolytic anemia
Hemangiosarcoma
Hemorrhagic gastroenteritis
Hepatic disease, especially hepatic lipidosis in cats.
Gastric dilatation-volvulus (GDV), mammary gland carcinoma and pancreatitis can also lead to DIC.

55
Q

The night after Valentines day, an obese male neutered Schnauzer is presented with a combination of vomiting, urinary accidents and diarrhea.

On physical exam there is muscle rigidity and chocolate wrappers visible in the diarrhea. The dog has a seizure on the exam table. An ECG is shown below.

T=103.2 F (39.6 C)..[N=99.5-102.5 F, N=37.2-39.2 C]
RR=40 brpm….[N= 15-34]
HR=136 bpm….[N=60-120]

Which one of the following choices is the best treatment plan?

A - Apomorphine, gastric lavage, Na-bicarbonate IV
B - Activated charcoal, sodium sulfate, methocarbamol IV
C - Diazepam, lidocaine drip
D - Phenobarbital, glycopyrrolate
E - Ipecac, theophylline, digoxin

A

C- Diazepam, lidocaine drip

Treat the ventricular-tachycardia with a lidocaine drip and address hyperactivity/seizures with diazepam (Valium ®) or methocarbamol. If response to diazepam inadequate, consider barbiturates like phenobarb or pentobarb (given S-L-O-W-L-Y).

This is a classic presentation of severe chocolate toxicity, due to toxic methylxanthine alkaloids (ie: theobromine, theophylline, caffeine). Common after Halloween, Christmas and Valentine’s Day holidays, see EXCITEMENT, SEIZURES, ARRHYTHMIAS. The seizure and ECG which shows ventricular tachycardia (V-tach) are the key pieces of information here:

Most chocolate toxicities are milder- hyperactivity, vomiting, urinating, diarrhea. If see in first hour BEFORE signs occur, (and dog not seizing), induce vomiting with Apomorphine (0.03 mg/kg IV), Ipecac (1-2 ml/kg PO) or hydrogen peroxide (1-5ml/kg PO).

If animal is sedated because of seizures, consider gastric lavage. If vomiting is controlled, give activated charcoal to decrease absorption of toxic alkaloids and an osmotic cathartic like Na-sulfate (1g/kg PO) to promote elimination.

Rx arrhythmias depending on type-usually a tachycardia in need of LIDOCAINE for V-tach or BETA-BLOCKERS (propranolol / metaprolol) for supraventricular tachycardias.

Chocolate toxicity from most toxic to least toxic: Cacao beans and baking chocolate are worse than semisweet chocolate which is worse than milk chocolate.

49 grams of baking chocolate (one 2 oz bar) can kill a 7 kg dog. It would take 420 grams of milk chocolate (about 8, 2 oz bars) bars to kill a 7 kg dog.

56
Q

A 3 year old Standardbred mare is presented with a 2-month history of exercise intolerance. Endoscopy shows the following image.

What is the diagnosis?

A - Laryngeal hemiplegia
B - Cleft palate
C - Dorsal displacement of soft palate
D - Pharyngeal lymphoid hyperplasia (PLH)
E - Epiglottic entrapment

A

This is dorsal displacement of the soft palate (DDSP). The caudal free margin of the soft palate moves dorsal to epiglottis, obstructing the airway and causing exercise intolerance. Rx conservatively, eliminating possible contributing diseases first (ie: rest, anti-inflammatories). Surgical treatments (Sternothyrohyoideus myectomy or soft palate resection) have mixed success rates around 50%.

Epiglottic entrapment is a big DDX for DDSP. Outline of the epiglottis can still be seen with epiglottic entrapment, UNlike DDSP.

Cleft palate is a newborn disease. See difficulty suckling, dysphagia, MILK DRIPPING from NOSTRILS. Euthanize if severe. Surgical closure if small.

Laryngeal hemiplegia (“Roarers”) present with inspiratory noise during exercise and exercise intolerance. Click here to see laryngeal hemiplegia. More than 90% occur on LEFT side. Rx is surgery.

Pharyngeal lymphoid hyperplasia (PLH) is common. Thought to be a normal immunologic event in younger horses.

57
Q

During a routine dental cleaning under isoflurane anesthesia on an 8-year old male neutered Doberman, the ECG monitor shows the following pattern.

The dog is stable and doing fine. What is this pattern?

A - Atrial fibrillation
B - Ventricular premature complexes
C - Atrioventricular (AV) block
D - Accelerated idioventricular rhythm
E - Sinus arrhythmia

A

These are ventricular premature complexes (VPCs).

You had better be thinking the dog is in early stages of Dilated Cardiomyopathy (DCM).

According to Merck, 9th ed. “…ventricular premature contractions on a routine ECG in a presumed healthy DOBERMAN Pinscher or BOXER is HIGHLY SUGGESTIVE of CARDIOMYOPATHY”.

Echocardiography is the test of choice for definitive diagnosis of DCM.

Remember this mnemonic for DCM breed predispositions: “DCM in a BOX” (ie: “D_obes, C_ockers, M_assive dogs (giant breeds), in a BOX_er”

ALWAYS FATAL. Death usually in 6 to 24 months after Dx. WORSE Prognosis in DOBES, generally survive less than 6 months from Dx.

58
Q

During a necropsy on a 4-year old Beagle who died suddenly, a severe right ventricular hypertrophy is noted.

Which condition is highest on the differential diagnosis list?

A - Mitral dysplasia
B - Tricuspid dysplasia
C - Aortic stenosis
D - Patent ductus arteriosus (PDA)
E - Pulmonic stenosis

A

Think pulmonic stenosis if you see RIGHT ventricular hypertrophy, because pulmonic valves blocks outflow from R ventricle (mostly dogs). Follow this link to see an angiogram image of pulmonic stenosis.

Heard as a systolic murmur heard most loudly LEFT chest between the 2nd and 4th intercostal (IC) space; genetic link in beagles. Predilection seen in many breeds, including Eng. Bullldog, Min. Schnauzer, Scotties, Chihuahuas, Cockers, Boxers.

Mitral stenosis can be confused with Aortic stenosis (also called sub-aortic stenosis [SAS]), a systolic murmur which may be heard most loudly on the left chest between the 2nd and 5th intercostal (IC) space or at the thoracic inlet (lateral to trachea).

Mitral dysplasia and other mitral valve problems are heard further back on left at 5th-6th IC. More common in CATS.

Tricuspid dysplasia is heard further back on RIGHT at 5th-6th IC. Uncommon. Expect a continuous murmur with PDA. Vast majority detected at first vaccination visit.

59
Q

A necropsy of 16 month old female guinea pig with a 2-week history of anorexia, diarrhea, lethargy and weakness reveals the following.

What is the diagnosis?

A - Ketosis
B - Vitamin C deficiency
C - Cholecalciferol toxicity
D - Metastatic calcification
E - Nutritional muscular dystrophy

A

Think scurvy (vitamin C deficiency) when you see hemorrhages SQ in a guinea pig. Look for swollen joints (source of lameness) and a hx of lameness, anorexia, diarrhea, weakness.

May be thin with a rough hair coat. See increased vulnerability to opportunistic infections, and sudden death.

Follow this link to see the original Merck image of scurvy in a guinea pig

Rx with daily vitamin C 5-10 mg/kg, PO or IM, for 1-2 wk.

AVOID multivitamins! May cause toxicity for overdose of other vitamins. Need minimum 10 mg vitamin C/day (30 mg/day for pregnant sows) in diet.

Metastatic calcification occurs mostly in MALES

Nutrional muscular dystrophy and vitamin E deficiency are the same thing- can present like vitamin C deficiency but without diarrhea.

Ketosis is mostly a disease of fat or pregnant animals

60
Q

A sandhill crane from a local zoo dies after a period of chronic weight loss and weakness. Nodular lesions are discovered in the liver and spleen.

Which one of the following choices is the most likely diagnosis?

A

Tuberculosis due to Mycobacterium avium is the most common cause of these findings. M. avium is challenging to control in zoos and poultry flocks, because treatment may not be effective. A 3-month quarantine of all new additions to the aviary is strongly recommended.

Histomoniasis can produce caseous lesions similar to that of tuberculosis, but usually only involves the ceca and liver of galliform birds, especially turkeys and chickens. Histomoniasis is also known as blackhead.

61
Q

This cow was found recumbent and blind, with dorsomedial strabismus, in tonic-clonic seizures that led to coma and death.

On necropsy, lesions shown below light up under ultraviolet light.

What caused this problem?

A - High sulfur diet
B - Rabies
C - Bovine spongiform encephalopathy
D - Urea toxicity
E - Thromboembolic meningoencephalitis (TEME)

A

This is the cerebrocortical necrosis of polioencephalomalacia (PEM), basically a nutritional disease. PEM is traditionally associated with LOW THIAMINE but increasingly associated with HIGH SULFUR diets.

PATHOGNOMONIC dorsomedial strabismus (“Stargazing”), history and cerebrocortical necrosis that lights up under UV light all suggest PEM.

62
Q

A 7 week old Pot-bellied Pig is presented with a posterior swelling that looks like this:

Which one of the following is the most likely clinical diagnosis?

A - Perineal hernia
B - Inguinal hernia
C - Testicular torsion
D - Cryptorchidism
E - Intersex syndrome

A

This is an inguinal hernia in a piglet.

A COMMON problem in pigs, Rx surgically: midline skin incision, cranial to scrotum; ligate and excise vas deferens, blood vessels.

BOTH inguinal ring areas should be closed to prevent herniation post-surgery. Removal of tunic, cremaster muscle, extra subQ tissue, with closure to obliterate empty space helps prevent seromas.

Follow this link to a good Merck diagram of inguinal hernia anatomy in a horse.

Intersex syndrome is described in pigs and goat (rare), but look for BOTH genitalia.

Perineal hernia more a problem of middle-aged pure bred dogs.

With cryptorchidism a testicle is RETAINED, not protruding.

63
Q

Which of the following nerves are targeted with a paravertebral block used to perform a standing laparotomy in a cow?

A - T13, L1, and L2
B - L1-3, and S1-5
C - L2, L3, S1, and S2
D - L1, L2, L3
E - L1, L2, and L4

A

The spinal nerves, T13, L1, and L2 must be blocked to completely desensitize the flank of a cow. The paravertebral (PV)nerve block targets these nerves. It can be performed via two techniques – the proximal or distal PV block.

The proximal block places local anesthetic in the space just caudal to the transverse processes of the vertebrae - T13, L1, and L2.

The distal block is placed at the ends of the transverse processes of the vertebrae – L1, L2, and L4 as the nerves gradually course caudally after they exit the spinal foramen.

Proper placement of the anesthetic results in warming of the skin from vasodilation, anesthesia of the skin and body wall, and a curvature of the spine in some cows.

The latter is caused by relaxation of the epaxial musculature on the affected side; the spine curves in a convex manner.

64
Q

A dog hit by a car presents with stiff hypertonic forelimbs and flaccid, paralyzed hindlimbs.

Where is the lesion likely to be?

A - Cranial cervical: C1-C5
B - Cervicothoracic: C6-T2
C - Thoracolumbar T3-L3
D - Lumbosacral L4-S3
E - Cannot tell without more information

A

Thoracolumbar T3-L3 . This is likely to be Schiff-Sherrington syndrome ie: severe spinal cord trauma T3-L3, with thoracic limb extensor rigidity and hind limb flaccid paralysis. Lesion is caudal to T2, but inhibitory neurons in lumbar spinal cord (especially L2-L4) affect neurons in the cervical intumescence ( C6-T2).

With severe trauma T2-T13, inhibitory pathways are interrupted; Cervical intumescence neurons are “released” and cause extensor hypertonia in the forelimbs.

Can localize by checking cutaneous trunci reflex-The lesion is usually 1-2 vertebrae cranial to the line of analgesia (where dog does not feel pinching skin).

65
Q

Pyrrolizidine alkaloid toxicity is caused by chronic ingestion of which one of the following plants?

A - Astragalus spp. (locoweed)
B - Lupinus spp. (lupine)
C - Nerium spp. (oleander)
D - Persea spp. (avocado)
E - Senecio spp. (ragwort)

A

Senecio spp. (ragwort).

Common plants containing pyrrolizidine alkaloids (PA) are: Senecio vulgaris, S. jacobea, Amsinckia intermedius, Heliotropium europaeum, Crotolaria spectabilis.

Although generally not palatable, livestock will eat these plants when baled in hay or on pasture when forage is scarce. Chronic ingestion allows accumulation of toxic levels of PA, resulting in hepatic fibrosis.

Poisoning is most common in horses and cattle, sheep and goats are more resistant to toxic effects.

Persea spp. (avocado leaves) contain persin. Signs of toxicity are: noninfectious mastitis, abrupt cessation of milkflow, heart failure.

Nerium spp (oleander) contain cardiac glycosides. Signs of toxicity are: sudden death, weakness, diarrhea, cardiac arrhythmias.

Lupinus spp. (lupine) contain alkaloids. Signs of toxicity are: birth defects (ingestion at 40-70 days in cattle), abortion, tremors, incoordination, head pressing, seizures.

Astragalus and Oxytropis spp. (locoweed) contain alkaloids (swainsonine). Signs of toxicity are: excitability, incoordination, difficulty eating, exaggerated mouth movements, depression.

66
Q

Which reportable condition can affect cattle, but mainly causes disease in sheep?

A - Anthrax
B - Rinderpest
C - Vesicular stomatitis
D - Malignant Catarrhal fever
E - Bluetongue

A

Bluetongue is almost exclusively a sheep disease, (but cattle and deer can get it).

Rinderpest mainly affects cattle and is reported to be eliminated as of October 2010 by the United Nation’s Global Rinderpest Eradication Program. Because it is a classic severe and reportable vesicular disease, vets will likely need to keep rinderpest on their mental DDX list for years to come.

Pseudorabies is basically a pig pathogen. Can affect cows, but horses (and humans) are resistant

Vesicular Stomatitis (VS) can occur in horses, pigs, cows. Remember the big 8 vesicular diseases: BVD, IBR, BPS, MCF, Bluetongue, VS, FMD, Rinderpest)

67
Q

You serologically test 100 Siberian box turtles for galloping halitosis.
27 turtles test seropositive and 73 test seronegative.

However, molecular testing reveals 3/27 of the seropositive turtles are disease free and 10/73 of the seronegative turtles are diseased.

The entire fur-bearing turtle industry depends on your answer: What is the Predictive Value Positive (PVP) of your serologic test?

A - 95%
B - 89%
C - 86%
D - 73%
E - 70%

A

It is 89%. Remember - you are comparing TWO TESTS here. PVP means “Of the turtles my test says are positive (27), how many are truly positive?” (27-3=24, this # goes in the “a” box)

Here is how you do it: First, draw a 2x2 table, and label the boxes a,b,c,d. PVP = a/(a+b). Click here to see a Basic 2X2 table. Now, add in the TOTAL number of animals (100), the total positive by YOUR test (27) and the total negative by YOUR test (73), like this diagram: 2x2 with totals.

Now the (slightly) tricky part. Add in the numbers that YOUR test got WRONG according to the gold standard test. (3 false pos in box b, 10 false neg in box c): Click here to see 2x2 with b and c cells.

Last, subtract to fill in your “d” box (73-10=63) and do the math to calculate PVP = a/(a+b)=24/27=0.89 or 89% : Click here to see the final 2x2 with all cells filled and PVP calculated.

FYI: You can calculate sensitivity a/(a+c), specificity d/(b+d), Predictive Value POS (PVP) a/(a+b) and Predictive Value NEG (PVN) d/(c+d) with the same 2x2 table.

68
Q

A flock from a turkey farm is presented with a mysterious illness.

Several dead birds are noted, mostly younger. Sick turkeys are listless, with drooping wings, unkempt feathers, yellow droppings. Sick older birds are emaciated.

Necropsy shows a yellowish green, caseous exudate in the ceca, cecal ulcerations and thickening of the cecal wall. A typical liver looks like the image below.

What is the diagnosis?

A

This is histomoniasis. The combination of characteristic “bulls-eye” lesions on liver and cecal changes are pathognomonic.
Caused by protozoan Histomonas meleagridis, transmitted in eggs of cecal nematode Heterakis gallinarum. Expect a depression/diarrhea presentation.

Expect to see more sudden death with necrotic enteritis caused by Clostridium perfringens. Follow this link to see the so-called “Turkish towel” intestinal pseudomembrane of necrotic enteritis.

Signs of avian spirochetosis are highly variable, may be absent: see listlessness, shivering, increased thirst, green/yellow diarrhea with increased urates early on.

Caused by a tick-borne Borrelia. Look for characteristic enlarged, mottled spleen with petechial hemorrhages, similar to Marble spleen disease of pheasants.
Expect depression, bloody droppings, substantial mortality with hemorrhagic enteritis of turkeys. Follow this link to see hemorrhagic intestines. Follow this link to see characteristic enlarged spleen.

Expect diarrheal presentation with coronaviral enteritis of turkeys but NOT the characteristic cecal/liver lesions described on necropsy above.

For a good visual resource key poultry diseases,see the Cornell Atlas of Avian Diseases

69
Q

Which lesions are most commonly associated with Marek’s disease?

A - Edematous facial swelling with sinusitis
B - Hemorrhagic skin lesions
C - Nerve enlargement
D - Wrinkled eggs
E - Blood in the trachea

A

Think of NERVE ENLARGEMENT with Marek’s disease; Also distortion of the pupil, enlargement of feather follicles (“skin leukosis”=condemnation of carcass). May see one leg forward, one leg back, a transient paralysis.

Think Infectious Laryngotracheitis (ILT) if there is blood occluding trachea on necropsy.

Remember wrinkled eggs go with Infectious Bronchitis.

Think Infectious Coryza with sinusitis, swelling under eyes.

70
Q

In North America, prophylactic tx of Hypoderma bovis infection in cattle should be completed by August or September in warmer areas and by October in colder areas for which of the following reasons?

A

Doramectin and ivermectin are systemically active against Hypoderma larvae when injected SC. Ivermectin is also available as an oral paste. The injectable and pour-on systemic treatments are approved for control of Hypoderma and other myiasis-causing flies in many countries.

Eprinomectin and moxidectin pour-on formulations are approved for treatment of both beef and dairy cattle. Otherwise, use of drugs for cattle grub control is prohibited in dairy animals of breeding age. Because residues may be present in cattle for varying periods after treatment, withdrawal times for all treatments must be observed.

In areas where Hypoderma spp are prevalent, cattle, especially calves, should be treated as soon as possible after the end of the heel fly season. They should not be treated later than 8–12 wk before the anticipated first appearance of grubs in the backs, because adverse reactions may occur when migrating larvae are killed.

71
Q

A sow at 110 days gestation is being examined because of the swollen vulva. A photograph is shown. What do you do?

A

No intervention- normal: 1-4 days until farrowing

72
Q

In the dog, the immune complexes formed during pyometra affect what?

A
73
Q
A
74
Q

6 month old male Belgian Malinois sudden onset of lameness and pain in the left forelimb. Previous episodes of lameness in both forelimbs. Temperature 39.7C. MIld bilateral swelling of the carpal joints, with extreme pain noted on palpation. What is the diagnosis?

A

Hypertrophic Osteodystrophy

The prognosis for any animal will naturally relate to the degree of affliction. Generally, the overall prognosis for affected animals is favorable, and relapses are rare.(10) Freedom from the disease appears certain only when an animal reaches skeletal maturity.

75
Q

Best treatment to confirm Addison’s disease?

A

ACTH Stimulation Testing

76
Q

3 month old Holstein calf

Progressive weakness of the HLs over the past week is now unable to stand

Temperature, pulse rate, and RR are WNL

Spastic hyperreflexia in the HLs and normal reflexes in the forelimbs

Tail/anal muscle tone appears normal to increased

Navel is swollen

CBC shows neutrophilia with a left shift

A

Vertebral body abscess

77
Q

A flock of 10,000 30 week old broiler breeders has an increase in mortality. On postmortem exam of the affected birds, a yellow, caseous material is seen in the wattle and peritoneums. Impression smears of lesions show gram-negative, nonspore-forming rods. What is the most likely diagnosis?

A

Fowl cholera (Pasteurellosis)

78
Q
A

Even if direct examination reveals fungal invasion of hair, fungal culture is needed to confirm the infection; the author still performs fungal culture for medical/legal reasons. It remains the ‘gold standard’ technique for diagnosis.1

79
Q

Bone spavin

A

Search Results

Bone spavin is the horseman’s term for osteoarthritis or degenerative joint disease of the hock joint. A very common cause of lameness in the adult performance horse, it generally occurs because of wear and tear or repetitive trauma to the hock joint.

80
Q

Green splint

A

A term for acute tearing of the ligament which holds the inside splint bone to the cannon bone

81
Q

Ringbone

A

Ringbone is exostosis (bone growth) in the pastern or coffin joint of a horse

82
Q

Thoroughpin

A
83
Q
A

The innocuous-sounding blemishes called “windpuffs” (or windgalls) are balloon-like fluid swellings of the deep digital flexor tendon sheath near the back of the fetlocks that makes the joint appear puffy in a concentrated area. Similar to thoroughpins in the hocks, these swellings do not emit heat or cause pain or lameness, but a soundness examination and possibly x-rays or an ultrasound by a veterinarian are often recommended, as windpuffs could signal changes in bones, joints, or soft tissues such as tendons and ligaments. They most commonly appear in the hind legs, but can also affect the forelegs, and usually indicate that the sheath or related structures have been injured, or stretched due to over-exertion, creating the extra fluid.

84
Q
A

Habronemiasis- larvae of the stomach worm migrate and emerge, creating granulomatous lesions, usually around the eye, male genitalia, or lower extremities. Inside the granulomas, you can find dead larvae

85
Q

The primary route by which cattle are infected with Brucella abortus?

A

n infected cow typically aborts only once after exposure; subsequent gestations and lactations appear normal. After exposure, cattle become bacteremic for a short period and develop agglutinins and other antibodies; some cattle resist infection, and a small percentage of infected cows spontaneously recover.

** Ingestion of infected aborted tissues and fluids

Natural transmission occurs by ingestion of organisms, which are present in large numbers in aborted fetuses, fetal membranes, and uterine discharges. Cattle may ingest contaminated feed and water or may lick contaminated genitals of other animals. Venereal transmission by infected bulls to susceptible cows appears to be rare. Transmission may occur by artificial insemination when Brucella-contaminated semen is deposited in the uterus but, reportedly, not when deposited in the midcervix. Brucellae may enter the body through mucous membranes, conjunctivae, wounds, or intact skin in both people and animals.

86
Q

Two 7 month old DSH kittens, one is smaller and mentally dull compared to the other. Deciduous teeth are present in the smaller kitten. What disorder?

A

Hypothyroidism

87
Q

A 9 yo DSH underwent splenectomy for systemic Mast Cell Disease three weeks ago. Despite full recovery from sx, he still has systemic signs of disease. Which medications would be most appropriate to alleviate this cat’s discomfort?

A

For the visceral and systemic form, signs will include: vomiting, enlarged lymph nodes, depression, anorexia and possibly ascites

** Antihistamine and gastrointestinal protectants

88
Q

A flock of 8 week old broiler chickens has a high condemnation rate at the processing plant because of lymphoid tumors in multiple organs. Which of the following is the most appropriate first question to ask the owner to determine the cause of the problem?

A

**Eradication of avian leukosis virus from primary breeding stocks is the most effective means to control avian leukosis virus infection and lymphoid leukosis in chickens

Tumors can be differentiated from those of Marek’s disease by gross and microscopic pathology and by molecular techniques that demonstrate the characteristic clonal integration of proviral DNA into the tumor cell genome with the associated disruption of the c-myc oncogene. Lymphoid leukosis cannot easily be differentiated from B-cell lymphomas caused by reticuloendotheliosis virus except by virologic assays; however, such tumors probably are extremely rare.

ELISA kits for detection of antibodies to avian leukosis virus subgroups A, B, and J are available commercially.

89
Q

A 4 yo MN DSH cat is evaluated for bad breath and decreased appetite for two weeks. Photograph shows cat’s mouth. What should be included as part of the diagnostic workup?

A

Feline Lymphocytic-Plasmacytic Gingivostomatitis (LPGS), by whichever of the many names it goes, is an oral inflammatory disease that, by some of the more generous estimates, will be encountered by owners of about half of all cats infected with FIV and experiencing related symptoms. It is the most common secondary problem associated with FIV infection and is almost universally characterized as difficult to treat.

** ELISA for FIV

90
Q
A

Pathological fracture

91
Q
A

Surgical intervention- UAP

92
Q
A

Erysipelothrix rhusiopathiae- Valvular endocarditis is most common in mature or young adult pigs and is frequently manifest by death, usually from embolism or cardiac insufficiency. Chronic arthritis, the most common form of chronic infection, produces mild to severe lameness. Affected joints may be difficult to detect initially but eventually become hot and painful to the touch and later visibly enlarged. Dark purple, necrotic skin lesions that commonly slough may be seen. Mortality in chronic cases is low, but growth rate is retarded.

93
Q
A

** Habronema

Clinical signs may be helpful in differential diagnosis. Thelaziasis tends to cause a chronic conjunctivitis. In horses, infective larvae of the stomach worms Draschia and Habronema spp may also produce ophthalmic lesions. These tend to occur near the medial canthus of the eyelid and are raised, ulcerative granulomas, often containing characteristic yellow, plaque-like “sulfur granules” 1-2 mm in diameter. Likewise, microfilariae of Onchocerca spp invade the eye and may result in ophthalmic manifestations. Small (<1 mm), raised, white nodules in the pigmented conjunctiva adjacent to the temporal limbus are pathognomonic of Onchocerca infection. Depigmentation of the bulbar conjunctiva in this area also frequently occurs. Other lesions of onchocerciasis involve the cornea and include edema and punctate or streaking opacities of the stroma, superficial erosions, and a wedge-shaped sclerosing keratitis emanating from the temporal limbus. Intraocular structures also may be affected by microfilariae of Onchocerca spp.

94
Q
A

Pericarditis

95
Q
A

E. 2339/2872

96
Q
A

variety of injectable ivermectin treatment regimens effective against both fur and ear mites have been reported, with the dosage of ivermectin 200–400 mcg/kg, SC, two or three treatments 10–21 days apart. Mites may also be treated with selamectin (20 mg topically every 7 days has been effective).

97
Q
A

ENTEROLITH

large, dense mineral opacity, consistent with an

enterolith

At Colorado State University equine veterinary care is delivered through the collaboration of three nationally recognized equine service centers:

(1) Colorado State University Veterinary Teaching Hospital Equine Service; (2) Colorado State University Equine Reproduction Laboratory; (3)

Colorado State University Orthopaedic Research Center. Equine treatment capabilities at CSU are at the forefront of equine veterinary medicine

through the shared expertise of these organizations.

The recurrence rate of enteroliths is unknown, but

dietary modifications such as avoiding alfalfa hay

are usually recommended.

Removing horses from dirt or gravel, which can

serve as a nidus is also a good practice.

TAKE HOME MESSAGE

Hand A Lee was never in California or Florida,

and although she did not fit many of the principal

predisposing factors usually seen in cases of colic

caused by enteroliths, her case was a classical

example of scenario encountered in such cases. This

can serve as a reminder that we have to always keep

our minds open to all possible causes of abdominal

pain in horses, regardless of predisposing factors.

This approach was instrumental in the diagnosis

of her problem, and ultimately resulted in her

full recovery.

DISCUSSION POINTS

Obstruction of the large colon with enteroliths is a

well-documented but not completely understood

cause of intestinal obstruction in the horse, and

the risk factors include the geographic location of

the animal, with California and Florida having the

highest prevalence for this cause of colic.

Other factors include breeds such as Arabians

and Arabian crosses, Morgans, American

Saddlebreds, Donkeys and Miniature Horses, and

also the consumption of alfalfa hay, and less than

50 percent of time spent outdoors.

Physical examination findings can vary depending

on where the stone is located inside the intestines

and whether or not the intestinal wall becomes

compromised by the pressure caused by

the stone.

Radiographs are a useful method to identify

enteroliths, although the sensitivity of this

technique can vary depending on the location of

the stone, its size and the size of the horse.

The prognosis for surgical removal of the stone

in cases that don’t have intestinal compromise is

usually excellent. However, local necrosis of the

intestine in a area that cannot be exteriorized is

associated with a grave prognosis.

98
Q
A

DDX Campylobacteriosis, Tritrichomonas

C. Examination of fetal tissues and uterine discharge for causative pathogens– better for Campy.

99
Q
A

A. Both animals are infested with Cheyletiella

100
Q
A

Respiratory acidosis

101
Q
A

B. Canine Scabies

102
Q
A

Pleuroperitoneal diaphragmatic hernia (in pleural space, not just abdominal viscera in the pericardial sac)

103
Q
A

Longitudinal incision distal to the foreign body (antimesenteric side)

104
Q
A

* Inadequate secretion of LH

To effectively detect estrus (heat) and successfully employ methods of estrus synchronization, the producer must have a basic understanding of the anatomy and reproductive physiology of sows and gilts. Figures 1 through 3 illustrate the endocrine system and anatomy that affect estrus. Estrus begins with the pituitary gland, which is located just below the brain and secretes several hormones into the blood stream, including luteinizing hormone (LH) and follicle-stimulating hormone (FSH). LH and FSH are called gonadotropins.

In prepubertal (immature) gilts, gonadotropin secretion is low but dramatically increases just prior to puberty (first estrus) at 6 to 8 months of age. During the two- to three-day period just prior to estrus, increasing blood levels of LH and FSH cause the follicles on each of the two ovaries to grow rapidly. These follicles in turn secrete increased levels of the hormone estradiol into the blood, causing the behavioral and physiological changes associated with estrus. Moreover, each follicle contains an ovum, which when released and fertilized by a sperm cell, develops into an embryo.

Rising concentrations of estradiol in the blood reach a threshold which triggers a massive release of LH from the pituitary gland around the onset of estrus. This LH surge stimulates ovulation, the release of ova from the follicles into the oviducts. Though the timing of ovulation is extremely variable, on average it occurs 40 hours after the onset of estrus. The sperm cells fertilize the ova in the oviducts, the tubes between the ovaries and the horns of the uterus. The fertilized eggs then progress to the uterus, implant, and further develop into embryos and then fetuses.

The sites on the ovaries from which ova are released subsequently form structures called corpora lutea that secrete another hormone, progesterone, into the blood. During the luteal phase of the estrous cycle (approximately day 4 to day 16), progesterone inhibits LH and FSH secretion from the pituitary gland, inhibiting follicular growth.

If the ova are not fertilized during estrus, or embryos do not implant in the uterus then beginning around day16 the uterus secrets the hormone prostaglandin-F2α (PGF2α) into the blood. PGF2α causes the regression or death of the corpora lutea and as a result, progesterone levels decline. Decreasing levels of progesterone allow LH and FSH levels to increase, follicles to grow, and estrus returns. Female swine display estrus at 18- to 22-day intervals throughout the year unless their cycling is interrupted by pregnancy and lactation, poor nutrition, disease, etc.

If fertilization occurs and pregnancy is initiated, then PGF2α is not released into the circulation. The corpora lutea are maintained and secrete high levels of progesterone into the blood stream throughout gestation. Progesterone is essential for maintenance of pregnancy. It inhibits follicular growth as well as uterine contractions.

Around day 114 of gestation, the uterus causes the corpora lutea to regress by releasing large amounts of PGF2α into the blood. Consequently, progesterone levels decrease, uterine contractions commence, and the fetuses are expelled.

During lactation, suckling by the pigs causes a suppression of LH and FSH secretion, keeping the ovaries devoid of large follicles. The removal of the suckling stimulus at weaning allows the secretion of gonadotropin to increase. Follicles grow rapidly and there is the corresponding rise in the circulating levels of estradiol. Sows return to estrus in four to seven days and estradiol elicits the surge of LH, causing ovulation.

105
Q
A

Panosteitis

106
Q
A

Pasteurella spp.

107
Q
A

Yersinia pestis

Y pestis is maintained in the environment in a natural cycle between susceptible rodent species and their associated fleas. Commonly affected rodent species include ground squirrels (Spermophilus spp) and wood rats (Neotoma spp). Cats and dogs are usually exposed to Y pestis by mucous membrane contact with secretions or tissues of an infected rodent or rabbit or by the bite of an infected flea. People are usually exposed by an infected flea bite but are sometimes exposed due to contact with infected animals or via respiratory droplet transmission from pneumonic cases. Risk factors for cats acquiring plague include hunting and eating rodents and rabbits, visiting an enzootic plague area, finding dead rodents around the yard or areas that the animal frequents, and exposure to infected fleas.

Plague must be differentiated from other bacterial infections, including tularemia (see Tularemia), abscesses due to wounds (cat fight bites), and staphylococcal and streptococcal infections. During acute illness, preferred antemortem samples for culture include whole blood, lymph node aspirates, swabs from draining lesions, and oropharyngeal swabs from cats with oral lesions or pneumonia. Diagnostic samples should be taken before antibiotics are administered. Y pestis cultures can take 48 hr for visible growth to develop. An air-dried glass slide smear of a bubo aspirate can be used for a fluorescent antibody test that detects the F1 antigen on Y pestis cells. This test can be performed in a matter of hours in an experienced laboratory and is both sensitive and specific.

Because of the rapid progression of this disease, treatment for suspected plague (and infection control practices) should be started before a definitive diagnosis is obtained. Streptomycin has been considered the drug of choice in human cases but is difficult to obtain and rarely used today. Gentamicin is currently used to treat most human plague cases and should be considered a suitable alternative choice in veterinary medicine for seriously ill animals, although it is not approved for this purpose. Animals with renal failure require adjusted dosages.

108
Q
A
109
Q
A

Yew - The bark and leaves of this very popular evergreen provided the basis for the cancer-treatment drug paclitaxel, but general ingestion of any part of the tree (except the flesh of the berry) can be very dangerous to animals. Horses have an especially low tolerance to yew. - central nervous system effects such as trembling, incoordination, and difficulty breathing, can cause significant gastrointestinal irritation and cardiac failure, which can result in death.

Morning Glory - can cause hallucinations, gastrointestinal upset, agitation, tremors, disorientation, ataxia, anorexia.

Oleander is pretty, and poisonous – can severely affect cats, dogs, and even horses. All parts contain a highly toxic cardiac glycoside and can cause a number of problems. - Colic, diarrhea (possibly bloody), sweating, incoordination, shallow/difficult breathing, muscle tremors, recumbence, and possibly death from cardiac failure.

uercus poisoning, although cattle and sheep are affected most often. Most species of oak (Quercus spp) found in Europe and North America are considered toxic. Clinical signs occur 3–7 days after consumption of large quantities of young oak leaves in the spring or green acorns in the fall. Fallen trees associated with a recent storm are often reported with outbreaks. Feed restriction before exposure plays a crucial role enhancing susceptibility. Mortality as high as 70% may be seen. Malformed calves and abortions have been reported in dams consuming acorns during the second trimester of pregnancy. The toxic principle, which appears to be pyrogallol, gallotannins, polyhydroxyphenolic compounds, or their metabolites produced by microbial hydrolysis in the rumen, binds and precipitates proteins, which results in GI and renal dysfunction.

Signs include anorexia, depression, emaciation, brisket edema, dehydration, rumen stasis, tenesmus, smell of ammonia on the breath, serous ocular or nasal discharge, polydipsia, polyuria, hematuria, icterus, and constipation followed by mucoid to hemorrhagic diarrhea. Renal insufficiency, which is evident 4–6 days after exposure, may be characterized by increased BUN and creatinine, proteinuria, glucosuria, hyperbilirubinuria, hyperphosphatemia, hypocalcemia, and urine with a low specific gravity. Pale swollen kidneys characterized by coagulative necrosis of the proximal convoluted tubular cells, perirenal edema, subcutaneous edema, ascites, and hydrothorax are common necropsy findings. Edema and subserosal petechial or ecchymotic hemorrhage of intestinal mucosa and ulceration of the esophagus and rumen may be seen. Evidence of hepatotoxicity characterized by increased liver enzymes may also be present.

Diagnosis is based on clinical findings, necropsy, history, and histopathologic examination of the kidney (ie, nephrosis).

110
Q

Red Maple Toxicity

A

An unidentified toxin with oxidant properties is present in the wilted or dried leaves of red maples. Only the red maple (Acer rubrum) and possibly closely related hybrids are known to be toxic. The toxin in red maples oxidizes hemoglobin with the formation of Heinz bodies, methemoglobinemia and subsequent hemolytic anemia. Poisoning is especially likely in the fall or following a storm when leaves of fallen branches become accessible to horses. The fresh green leaves apparently are not toxic, but once dried they may remain toxic for up to 30 days. The bark from red maple trees is also toxic. Fatal poisoning of ponies fed 3.0 kg of dried red maple leaves occurred in 1 to 5 days. Doses of half this amount will induce formation of Heinz bodies. Other maple species including the sugar maple (Acer saccharum)have been suspected of causing poisoning in horses.

Treatment

Horses should be denied access to red maple leaves. Blood transfusions may be given as necessary. Administration of intravenous fluids are of benefit in preventing dehydration and loss of kidney function. Methylene blue advocated for the treatment of erythrocyte oxidant damage should be used with extreme caution in horses, and it is contraindicated if Heinz bodies are already formed as methylene blue induces Heinz body formation. The dosage of methylene blue should not exceed 8 mg/kg body weight and should be administered slowly intravenously as a 1% solution. High doses of vitamin C given intravenously has been shown to be beneficial in preventing the oxidative damage from the red maple toxiin.

Cardiovascular system

Acute hemolytic anemia. Cyanosis and brown discoloration of the blood due to methemoglobin.

Respiratory System

Increased respiratory rate.

Nervous System

Depression.

Renal System

Dark red-brown urine due to hemoglobinurea and methemoglobinurea.

Reproductive System

Pregnant mares may abort without showing signs of hemolytic anemia.

Hepatic System

Icterus (jaundice)

Diagnosis

Hemolytic anemia, with Heinz body formation, with evidence that horse has had access to wilted or dried red maple leaves

Special Notes

The prognosis is always guarded to poor for horses with red maple poisoning because of the rapid development of intravascular hemolysis, coagulopathy, hemoglobin nephropathy and vascular thrombosis. Red maple trees should not be planted in or around horse enclosures. Regular removal of the dried maple leaves in the Fall to prevent their accumulation in horse enclosures is recommended.

111
Q
A

63/963

112
Q
A

Mitral valve insufficiency

113
Q
A

Specificity

2339/2872

114
Q
A

Gastrostomy tube because you would have to place the orogastric tube each time

115
Q
A

Analysis of a blood sample for heavy metals

116
Q
A

** Prognosis is good with appropriate treatment

Aggressive IV fluid therapy is the mainstay of treatment. The rate of isotonic fluid administration is based on patient perfusion, degree of dehydration, and ongoing losses. Dogs markedly hypoproteinemic or in shock may benefit from synthetic or natural colloid (stored or fresh frozen plasma) therapy. Parenteral antibiotics effective against Clostridium spp (eg, ampicillin 22 mg/kg, IV, tid-qid, or metronidazole 7.5 mg/kg, IV, bid) and to decrease the potential for sepsis secondary to intestinal bacterial translocation are indicated. Additional antibiotic coverage for gram-negative bacteria (eg, enrofloxacin 5–10 mg/kg/day, IV) is indicated in animals with sepsis or neutropenia.

117
Q
A

DDFT

118
Q
A

Zinc Deficiency

119
Q
A

D. Multifocal hemorrhage and necrosis in multiple organs in neonatal puppies

Diagnosis:

In systemically affected puppies, CHV infection may be confused with infectious canine hepatitis (see Infectious Canine Hepatitis), but it is not accompanied by the thickened, edematous gallbladder often associated with the latter. The focal areas of necrosis and hemorrhage, especially those that are seen in the kidneys, distinguish it from hepatitis and neosporosis (see Neosporosis). CHV causes serious disease only in very young puppies. The rapid death and characteristic lesions distinguish it from canine distemper (see Canine Distemper).

Treatment:

Therapy is typically unrewarding in systemically affected puppies, and the prognosis for puppies that do survive is guarded because damage to lymphoid organs, brain, kidneys, and liver may be irreparable. Before onset of clinical signs in littermates or other nearby puppies, rearing in incubators at an increased temperature (95°F [35°C], 50% relative humidity), and/or passive immunization with intraperitoneal serum may reduce losses within an exposed litter. Limited studies with antiviral agents such as vidarabine are inconclusive, but immediate recognition and treatment would be needed to have any possibility of success.

Adult dogs with ocular, respiratory, or genital disease often experience mild and self-limiting signs. Ophthalmic antiviral cidofovir (0.5% bid) has been used successfully in one reported case of primary ocular infection in an adult dog and may be useful for persistent or painful ocular lesions.

120
Q
A

A.

121
Q
A

C. ELISA

122
Q
A

D. Pericardial effusion

This is technically not an arrhythmia, but occurs secondary to some underlying pathology. Most commonly it is associated with pericardial effusion, but can also occur secondary to pleural effusion and atrial tachycardia. In the instance of pericardial effusion, the heart tends to move within the fluid with each contraction. “When the motion of the heart within (a) pericardial effusion is increased markedly and, in particular, when this is accompanied by tachycardia (also secondary to some underlying pathology), the heart may not have returned to its previous position by the time the next cardiac cycle commences. With each depolarization the heart is in a slightly different position, and the electrocardiographic QRS complex also varies with alternating heights of R waves on the electorcardiogram, i.e., electrical alternans.” Because electrical alternans is a secondary electrocardiographic finding, it is necessary to treat the underlying cause (pericardiocentesis is often indicated).

123
Q
A

* Vagal indigestion gradual develoopment of abdominal distension secondary to rumenoreticular distension.

* Diseases that result in injury, inflammation, or pressure on the vagus nerve can result in clinical signs of vagal indigestion syndrome. e.g. Traumatic reticuloperitonitis

* Conditions resulting in mechanical obstructionof the cardia or reticulo-omasal orifice (e.g. papillomas or ingested placenta) can also cause if ruminoreticular distension is present and the condition is subacute to chronic

* Type I- failure or eructation or free gas bloat- can be caused by pharyngeal trauma

* Type II failure of omasal transport- prevents ingesta from passing through the omasum into the abomasum. Adhesions and abscesses are the most common cause of omasal transport and are located on the right or medial wall of the reticulum near the route of the vagus nerve. Usually due to traumatic reticuloperitonitis!!!!

* Type III secondary abomasal impaction- primary abomasal impaction develops due to feeding of dry, course roughage, such as straw, with restricted access to water usually cold temps. Secondary occurs after traumatic reticuloperitonitis or as a sequela to abomasal volvulus. The abomasum has difficulty emptying larger particles of food because of increased viscosity, they accumulate.

Type IV indigestion of late gestation (types I and IV are rare)

124
Q
A

Infectious bronchitis (IB)

This is an acute, highly contagious disease caused by a coronavirus, and characterised by tracheal rales, coughing and sneezing. Nasal discharge can occur in young chicks and a drop in egg production is seen in laying flocks. The consequences on growth performance and egg production make IB a disease of great economic importance.

125
Q
A

E. Phosphatidylcholine, prednisolone, ursodiol

Phosphatidylcholine- It has been shown that phosphatidylcholine (PC) extracted from egg yolk possesses a variety of biological activities, such as anti-inflammatory and anti-oxidant effects, and prevents oxidative stress.

Ursodiol, also known as ursodeoxycholic acid, is a naturally occurring bile acid. It suppresses hepatic synthesis and secretion of cholesterol and decreases intestinal absorption of cholesterol. Reducing cholesterol saturation allows solubilization of cholesterol-containing gallstones. Ursodiol also increases bile flow and reduces the hepatotoxic effect of bile salts by decreasing their detergent action. In small animals, ursodiol may be useful in treatment of cholesterol-containing gallstones, idiopathic hepatic lipidosis, and chronic active hepatitis. The dosage in dogs and cats is 15 mg/kg/day, PO.

126
Q
A

D

Oslerus osleri

Lungworm of Dogs

Adult Parasite:

The adult worms measure 5 mm (male) to 12 mm (female) (see picture at left).

Oslerus osleri in tracheal nodes

Hosts:

Definitive - Dog and dingo.

Life Cycle:

The first stage-larvae pass in the feces or in vomitus. The dog eats the feces or vomitus containing the first stage-larvae. The larvae make their way to the lung where they develop to adults in nodules in the trachea and bronchi. The females lay thin-walled eggs which hatch in the lung. The larvae are carried up the trachea and are swallowed. The prepatent period is about 10 weeks.

** Treatment: Febendazole or Ivermectin

127
Q
A

Nasal aspergillosis because of the distribution of the lesion– destruction of nasal turbinates. Occasional bloody discharge.

128
Q
A

Primary photosensitization

129
Q
A

staph aureus

130
Q
A

** Septic Metritis- enlarged uterus, fetid watery red-brownish uterine discharge, signs of systemic illness, and fever, within 21 days in milk.

Endometritis- purulent uterine discharge after 21 DIM or mucopurulent after 26 DIM

Pyometra- accum. purulent or mucopurulent exudate in the uterus, most do not exhibit signs of systemic illness

131
Q

Tongue paralysis horse??

A

While rabies may cause paralysis of the tongue and difficulty swallowing, the recent history of castration in this camel makes tetanus most likely.

132
Q
A

Mycoplasma mycoides mycoides (large colony variant) or other Mycoplasma spp may show severe lameness with multiple hot swollen joints, weight loss, pyrexia, and poor coats. Some have diarrhea, and some have increased lung sounds and respiratory rates. Affected kids are generally 2–4 wk old. Morbidity and mortality rates of 90% and 30%, respectively, have been reported. Adult does with Mycoplasma infection may have mastitis and polyarthritis. Treatment is with tetracycline, tylosin, or tiamulin, but prognosis for complete recovery is guarded.

133
Q
A

B 21-22 (same as cows, goats, pigs)

sheep- 17 days

dog 2-3 weeks

cat 1-7 days

134
Q
A

ADVERTISEMENT

Feline corneal diseases: Herpes and more (Proceedings)

Aug 01, 2009

By B. Keith Collins, DVM, MS, DACVO

CVC IN KANSAS CITY PROCEEDINGS

1234

Next

Herpesvirus infections

Ocular disease due to feline herpesvirus (FHV) is common. It is estimated that 80% of cats are latently infected with the virus, and approximately 40% of these cats will suffer recrudescent infection in later life. These estimates are based on data that is several decades old, and the actual percentages may be higher. FHV affects cats of all ages, but the initial (or primary) infection usually occurs in neonatal and adolescent cats. Symptoms include bilateral conjunctivitis, respiratory disease, and fever. Most cats recover from the primary infection in 7-10 days without specific antiviral treatment, but neonatal cats are more likely to suffer serious corneal and conjunctival scarring. Ocular infection can be unilateral or bilateral and with or without respiratory signs. Unilateral conjunctivitis or ulcerative keratitis in the absence of respiratory signs is common in adult cats with recrudescent infection. Additional conditions with possible association to FHV include non-ulcerative (or stromal) keratitis, conjunctival and corneal scarring (symblepharon), corneal sequestrum, eosinophilic keratitis, keratoconjunctivitis sicca (KCS), blocked nasolacrimal duct, and possibly uveitis. The remainder of this discussion will focus on corneal diseases. Stress appears to be an important factor in ocular FHV infections. “Stressful” events may include topical or systemic corticosteroids, concurrent systemic disease, anesthesia, hospitalization, acquisition of a new cat, or extended owner absences (e.g., vacation).

Herpesvirus keratitis

FHV keratitis can be ulcerative or non-ulcerative, but the ulcerative form is most common. The ulcerative keratitis is due to direct cytopathic effects of the virus. The non-ulcerative form (stromal keratitis) is primarily immune-mediated, occurs in response to viral antigen, and is characterized histologically by lymphocytic infiltrates. Corneal ulcers are most often superficial and irregular or geographic (map-like). However, faint linear to tree-branching ulcers, or dendritic ulcers, are considered pathognomonic of FHV. Dendritic ulcers can be detected with fluorescein stain using magnification and a cobalt blue light, but they may be easier to detect using rose Bengal stain. Ulcers resulting from especially virulent strains of FHV, or secondarily infected with bacteria, can quickly deteriorate. Conjunctival graft or corneo-conjunctival transposition surgery may be required for a deep stromal ulcer or descemetocele. Any cat with a corneal ulcer of undetermined cause should be promptly treated with an antiviral agent.

The non-ulcerative or stromal keratitis is characterized by circumcorneal to diffuse corneal stromal opacity and blood vessels. Because the stromal keratitis is immune-mediated, improvement may occur with judicious application of topical steroids. However, topical steroids should be used only if the cornea is negative to fluorescein stain and if prior antiviral treatments have been ineffective. Then, they should only be used in conjunction with an antiviral agent. Caution is advised when contemplating steroid use in a cat with suspected FHV, and frequent examinations (at least initially) are prudent if they are used.

135
Q
A

Cholecalciferol toxicosis is characterized by hyperphosphatemia and hypercalcemia, leading to renal failure, cardiac abnormalities, hypertension, CNS depression, anorexia, vomiting, diarrhea, and lethargy. The increased calcium and phosphorus can lead to calcification of soft tissue, notably the highly vascular areas of kidneys and lungs, as well as within the walls of the great blood vessels.

Clinical signs generally develop within 18–36 hr of ingestion; initial signs can include depression, anorexia, polyuria, and polydipsia. The serum phosphorus more commonly rises first, at ~12–24 hr after ingestion, with serum calcium levels rising within another 12–24 hr. Nausea, vomiting, hematemesis, and depression are common as the clinical signs progress. It is important to obtain a baseline biochemistry profile as early as possible after the exposure, so that each animal can be monitored based on individual values.

Ingestion of vitamin D3 at >0.1 mg/kg may require decontamination (induction of emesis and administration of activated charcoal) and monitoring of serum calcium, phosphorus, and renal values. Emesis can be induced within 2 hr of exposure with 3% hydrogen peroxide or apomorphine in dogs and xylazine in cats. Activated charcoal at 1–2 g/kg is an appropriate initial dose for decontamination, and a second half dose after ~6–8 hr may be helpful. In addition, use of cholestyramine, a bile acid sequestrant, may be useful to decrease the body burden of vitamin D3 that undergoes enterohepatic recirculation with bile acids. However, the efficacy of cholestyramine to reduce vitamin D3 levels in dogs has not been determined. The recommended dosage is 0.3–0.5 g/kg, dissolved in liquid and administered orally every 6–8 hr for 3–5 days, depending on the initial dose of cholecalciferol ingested. Premature initiation of calciuresis (see below) may disrupt normal calcium-phosphorus metabolism, triggering osteoclasts to move additional calcium into the blood stream, artificially increasing serum calcium and phosphorus levels and mimicking vitamin D3 toxicosis.

136
Q
A

C

137
Q
A

PDA

138
Q
A

D.

Diagnosis of MCF is based on clinical signs, gross and histologic lesions, and laboratory confirmation. Primary differential diagnoses include bovine viral diarrhea/mucosal disease, rinderpest, infectious bovine rhinotracheitis, and East Coast fever (theileriosis). When CNS involvement is prominent, MCF can resemble rabies and the tickborne encephalitides. A history of contact with a carrier species (sheep, goats, or wildebeest) can be helpful, although recrudescent cases can be seen without such a history. Reliable and specific laboratory assays for antibody and for viral DNA are available. The test of choice for clinical diagnosis is PCR to detect viral DNA. Preferred tissues for testing are anticoagulated blood, kidney, intestinal wall, lymph node, and brain.

Serology is used to survey healthy animals and is indicative only of infection—latent infection among susceptible animals may render serology alone inconclusive evidence of current disease. Several seroassays are available, including viral neutralization, immunoperoxidase, immunofluorescence, and ELISA. The polyclonal assays are hampered by cross-reactivity. The monoclonal-based competitive ELISA is currently the most specific and detects antibody against all of the known MCF group viruses. Only PCR can discriminate between the different viruses.

Treatment and Control:

The prognosis is grave. No treatment has been found to provide any consistent benefit. Stress reduction of subclinical or mildly affected animals is indicated. No vaccine is currently available. Sheep can be produced that are free of virus by early weaning and isolation. The only other effective control strategy is separation of carriers from susceptible species. When large numbers of potent shedders are present, such as in lamb feedlots, distances >1 km may be necessary to protect highly susceptible species such as bison.

Overview of Hemorrhagic Septicemia

Was This Page Helpful?

Overview of Nairobi Sheep Disease

Also of Interest

Test your knowledge

Anaerobic clostridial bacteria release toxins that can cause severe localized and systemic disease, many of which can be fatal if untreated. Which one of the following clostridial diseases results in intravascular hemolysis, hemolytic anemia, and hemoglobinuria in cattle?

Botulism Enterotoxemia Malignant edema Red water disease

Heartwater case, signs and treatment, goat

Goat showing signs of heartwater being treated with IV oxytetracycline.

More Videos

MSD and the MSD Veterinary Manual

Merck & Co., Inc., Kenilworth, NJ, USA (known as MSD outside of the US and Canada) is a global healthcare leader working to help the world be well. From developing new therapies that treat and prevent disease to helping people in need, we are committed to improving health and well-being around the world. The Veterinary Manual was first published in 1955 as a service to the community. The legacy of this great resource continues as the Merck Veterinary Manual in the US and Canada and the MSD Veterinary Manual outside of North America.

MsdManuals

Disclaimer

Permissions

Privacy

Licensing

Terms of use

Contact Us

Human Health Manuals

Glossary

© 2016 Merck Sharp & Dohme Corp., a subsidiary of Merck & Co., Inc., Kenilworth, NJ, USA

The prognosis is grave. No treatment has been found to provide any consistent benefit. Stress reduction of subclinical or mildly affected animals is indicated. No vaccine is currently available. Sheep can be produced that are free of virus by early weaning and isolation. The only other effective control strategy is separation of carriers from susceptible species. When large numbers of potent shedders are present, such as in lamb feedlots, distances >1 km may be necessary to protect highly susceptible species such as bison.

139
Q
A

Anaplasma marginale- would be in WBCs

Babesia in a cat- does not usually have fever (not jaundice either)

Cytauxzoon- WBCs as well

** Mycoplasma haemofelis

140
Q
A

** Very long tubular horns

Diffuse: Almost the entire surface of the allantochorion is involved in formation of the placenta. Seen in horses and pigs.

Cotyledonary: Multiple, discrete areas of attachment called cotyledons are formed by interaction of patches of allantochorion with endometrium. The fetal portions of this type of placenta are called cotyledons, the maternal contact sites (caruncles), and the cotyledon-caruncle complex a placentome. This type of placentation is observed in ruminants.

Zonary: The placenta takes the form of a complete or incomplete band of tissue surrounding the fetus. Seen in carnivores like dogs and cats, seals, bears, and elephants.

Discoid: A single placenta is formed and is discoid in shape. Seen in primates and rodents.

141
Q
A

Red maple (Acer rubrum)- methaemoglobinaemia- brown blood

142
Q
A

Oxytetracyclines

In animals <1 yr old anaplasmosis is usually subclinical, in yearlings and 2-yr-olds it is moderately severe, and in older cattle it is severe and often fatal. Anaplasmosis is characterized by progressive anemia due to extravascular destruction of infected and uninfected erythrocytes. The prepatent period of A marginale is directly related to the infective dose and typically ranges from 15–36 days (although it may be as long as 100 days). After the prepatent period, peracute (most severe but rare), acute, or chronic anaplasmosis may follow. Rickettsemia approximately doubles every 24 hr during the exponential growth phase. Generally, 10%–30% of erythrocytes are infected at peak rickettsemia, although this figure may be as high as 65%. RBC count, PCV, and hemoglobin values are all severely reduced. Macrocytic anemia with circulating reticulocytes may be present late in the disease.

Animals with peracute infections succumb within a few hours of the onset of clinical signs. Acutely infected animals lose condition rapidly. Milk production falls. Inappetence, loss of coordination, breathlessness when exerted, and a rapid bounding pulse are usually evident in the late stages. The urine may be brown but, in contrast to babesiosis, hemoglobinuria does not occur. A transient febrile response, with the body temperature rarely exceeding 106°F (41°C) occurs at about the time of peak rickettsemia. Mucous membranes appear pale and then yellow. Pregnant cows may abort. Surviving cattle convalesce over several weeks, during which hematologic parameters gradually return to normal.

Bos indicus breeds of cattle appear to possess a greater resistance to A marginale infection than B taurus breeds, but variation of resistance of individuals within breeds of both species occurs. Differences in virulence between Anaplasma strains and the level and duration of the rickettsemia also play a role in severity of clinical manifestations.

143
Q
A

Mitral stenosis

144
Q
A

A cat that is normally hydrated and has a USG of less than 1.007, a cat that is dehydrated– it would be dangerous– already confirmed anyway

145
Q
A

The teeth are longest on the buccal surface of the upper arcade and the lingual surface of the lower arcade

146
Q
A

Leptospira spp.

147
Q
A

A. Serological ID and vaccination of infected animals

148
Q
A

Toxascaris leonina (though rare)

149
Q
A

Providing mineral supplementation

150
Q
A

Abomasum- Ostertagia

151
Q
A

Nephrotoxicity– A– BUN, Serum Creatinine and USG

152
Q
A

Lasalocid

Lambs 1–6 mo old in lambing pens, intensive grazing areas, and feedlots are at greatest risk as a result of shipping, ration change, crowding stress, severe weather, and contamination of the environment with oocysts from ewes or other lambs. Because occurrence of coccidiosis under these management systems often becomes so predictable, coccidiostats should be administered prophylactically for 28 consecutive days beginning a few days after lambs are introduced into the environment. A concentrated ration containing monensin at 15 g/tonne can be fed to ewes from 4 wk before lambing until weaning, and to lambs from 4–20 wk of age. The toxic level of monensin for lambs is 4 mg/kg. Lasalocid (15–70 mg/head/day, depending on body wt) may be effective. A combination of monensin and lasalocid at 22 and 100 mg/kg of diet, respectively, is an effective prophylactic against naturally occurring coccidiosis in early weaned lambs under feedlot conditions.

153
Q
A

Refer for cardiac catheterization

Because of the high risk of CHF early in life for animals with a PDA, closure is generally recommended. There are two major treatment options for PDA closure: interventional transcatheter occlusion and surgical ligation. Catheter-based occlusion is minimally invasive and typically involves placement of an occlusion device through a peripheral vessel (most commonly the femoral artery). The Amplatz canine ductal occluder is used most often and is highly successful with minimal complications. The use of other occlusion devices such as Gianturco coils and vascular plugs has also been described. The major limitations to a transcatheter approach are ductal size and patient size. Surgical ligation of the ductus is also usually curative and considered most often for small dogs and cats. If present, CHF should be medically managed before anesthesia and surgery are performed.

154
Q
A

Uterine biopsy

155
Q
A

Injection with iron dextran of newborn piglets

156
Q
A

Normal Finding

157
Q
A

Confirm by Western blot or immunofluorescent antibody test

With the currently available ELISA in-house test kits, sensitivity is high. However, these tests cannot distinguish between antibodies produced in response to vaccination with FIV vaccine and those produced by natural infection. An ELISA test that appears to successfully discriminate between antibodies produced in response to vaccination and those produced in natural infections has been developed but is not available for commercial use. A reasonable approach is to use the in-house test as a screening test. A negative result is a fairly reliable indicator that the cat is not infected. Positive results, especially in asymptomatic cats, should be confirmed by another test such as Western blot if available. Kittens born to vaccinated queens are antibody-positive for a variable length of time.

158
Q
A

Beta lacatams

The vast majority of abscesses result in a large quantity of Staphylococcus spp bacteria, but other bacteria may be involved, such as Escherichia coli, Streptococcus spp, Pseudomonas spp, Mycoplasma spp and Mycoplasma-like organisms (L-forms), Pasteurella multocida, Corynebacterium spp, Actinomyces spp and Nocardia spp.

Obligate anaerobes, including Bacteroides spp, Clostridium spp, Eubacterium, Fusobacterium SPP, and Peptostreptococcus SPP may also be involved[1].

FIV is the leading secondary complication in cats which exhibit inter-cat aggression, as well as the risk of secondary bacteremia.

Fungal infections can cause abscess, including Cryptococcus spp and Coccidioides spp.

159
Q
A

Toxocara canis or cati

160
Q
A

Cephalexin

161
Q
A

*If second stage of labor lasts longer than 4 hours

Bitches typically enter stage I labor within 24 hr of a decline in serum progesterone to <2–5 ng/mL, which develops in conjunction with increased circulating prostaglandins and is commonly associated with a transient drop in body temperature (<99°F [37.2°C]). Monitoring serial progesterone levels for impending labor is problematic because in-house kits enabling rapid results are inherently inaccurate between 2 and 5 ng/mL. Commercial laboratories offering quantitative progesterone by radioimmunoassay or hemiluminescence typically have a 12–24 hr turnaround time, which is not rapid enough to make decisions about immediate obstetric intervention. Progesterone levels can drop rapidly in a matter of hours. If progesterone is <2 ng/mL, gestation is likely at term and labor pending. Clearly, it is beneficial to obtain information about ovulation timing, minimally by determining the onset of cytologic diestrus, to evaluate length of gestation at term.

162
Q
A

Application of positive pressure ventilation

163
Q
A

A. 0.9% Sodium chloride with added KCl

A mild metabolic alkalosis with hypochloremia and hypokalemia are common

The hypochloremic metabolic alkalosis is due to abomasal hypomotility, continued secretion of hydrochloric acid into the abomasum, and the partial abomasal outflow obstruction, with sequestration of chloride in the abomasum and reflux into the rumen. Hypokalemia is due to decreased intake of feeds high in potassium, sequestration of potassium in the abomasum, and dehydration. Secondary ketosis is common and may be complicated by development of hepatic lipidosis (fatty liver disease; see Fatty Liver Disease of Cattle).

164
Q
A

VENTRAL MEATUS

165
Q
A

Interdigital dermatitis- Currently, the treatment of choice consists of external application of 10% w/v zinc sulfate disinfectants via a footbath or aerosol.

D.

166
Q
A

Allergic contact

167
Q
A

A.

The discovery of Strep. ag. or Mycoplasma

in bulk tank milk should be followed as soon

as possible by confirmatory testing. Sample

s should be collected from all cows with

clinical mastitis; “string” samples should be

collected from all milking strings or pens,

including the hospital pen; and another bul

k tank sample should be collected. These

samples will reveal the extent of spread of

these contagious pathogens, which generally

occurs in two distinct scenarios.

168
Q
A

Lyme Disease (Borreliosis, Borrelia burgdorferi)

Acute onset of lameness (the most common and characteristic observable symptom; polyarthritis; usually lasts only a few days but frequently recurs; may come and go; may be persistent; can become chronic)

Shifting leg lameness (limping changes from leg to leg)

Swollen joints (especially the hocks of the hind legs and the “wrists” [carpi] of the front legs)

Warmness around joints

Painful joints (especially when palpated/moved or manipulated manually)

Reluctance to rise

Reluctance to walk

Exercise intolerance

Stiff, stilted gait

Arched back (when standing or walking)

Fever (variable; usually low-grade but can be elevated)

Enlarged lymph nodes (lymphadenopathy)

Lack of appetite (inappetence; anorexia)

Weight loss

Depression

Weakness

Lethargy

Kidney disease. Severe renal disease has been widely reported in dogs with Lyme disease; when this unique form of kidney damage occurs, it usually presents with progressive vomiting, diarrhea, anorexia, weight loss, increased water intake (polydipsia), increased urine output (polyuria), swelling of the extremities (peripheral edema) and, unfortunately, death

Cardiac (heart) abnormalities (uncommon; severe when present; usually sudden in onset; can be fatal)

Neurological abnormalities (central nervous system consequences of Lyme disease are rare; seizures, facial paralysis and behavioral changes such as aggression have been attributed to Lyme disease in dogs)

Vomiting

Diarrhea

DDX Anaplasmosis– Tetracycline treatment also (doxy)

169
Q
A

Leptospirosis (DDX Parvovirus)

170
Q
A

** Doxycycline

Ehrlichiosis- monocytes- transmitted by R. sanguineus-worldwide- lymphoreticular system… acute cases reticuloendothelial hyperplasia, fever, general lymphadenopathy, splenomegaly, and thrombocytopenia… chronic– certain breeds predisposed GSDs– splenomegaly, glomerulonephritis, renal failure, interstitial pneumonitis, anterior uveitis, and meningitis, cerebellar ataxia, depression, paresis, hyperaesthesia.

Diagnosis- clin signs, positive serum indirect fluorescent antibody (IFA) titer. PCR

Treatmnet- Doxycycline

171
Q
A

E. Residue many months

172
Q
A

* Pyogranulomatous meningoencephalitis

Feline infectious peritonitis (FIP) is an immune-mediated disease triggered by infection with a feline coronavirus (FCoV). FCoV belongs to the family Coronaviridae, a group of enveloped, positive-stranded RNA viruses frequently found in cats. Coronavirus-specific antibodies are present in as many as 90% of cats in catteries and in as many as 50% of those in single-cat households. However, <5% of FCoV-infected cats develop FIP in multicat households.

FCoV and FIP are a major problem in multicat households. The virus is endemic in environments in which many cats are kept together in a confined space (eg, catteries, shelters, pet stores). FCoV is found less commonly in free-roaming community cats, because they do not typically use the same locations to bury their feces; shared litter boxes are a major source of transmission in multicat households.

FCoV is shed mainly in the feces. Infection is generally via the oronasal route. After natural infection, cats begin to shed virus in feces within 1 wk. In very early infection, it may be found in saliva, respiratory secretions, and urine. When naive cats in multicat households first encounter FCoV, it is likely that all will become infected (and develop antibodies); most will shed virus intermittently for a period of weeks or months. Some cats become chronic FCoV shedders, providing a continual source for reinfection of other cats. Cats that are antibody-negative are very unlikely to shed, whereas approximately one-third of all FCoV antibody-positive cats shed virus. Cats with high antibody titers are more likely to shed FCoV. They also are more likely to shed consistently and higher amounts of the virus. Most cats with FIP also shed non-mutated FCoV; however, the virus load in feces seems to decrease after a cat has developed FIP.

Feline Coronavirus Infection:

FCoV infection can cause a transient and clinically mild diarrhea and/or vomiting due to replication of FCoV in enterocytes. Kittens infected with FCoV may have a history of stunted growth or upper respiratory tract signs. Occasionally, the virus may cause severe diarrhea with weight loss, which may be unresponsive to treatment and continue for months. However, most FCoV-infected cats do not show clinical signs.

Feline Infectious Peritonitis:

Clinical signs of FIP vary depending on organ involvement. Many organs, including the liver, kidneys, pancreas, CNS, and eyes, can be involved. The clinical signs and pathologic findings are a consequence of the vasculitis and, less commonly, organ failure resultant from damage to the blood vessels that supply them. In all cats with nonspecific clinical signs, such as chronic weight loss or fever of unknown origin resistant to antibiotic treatment or recurrent in nature, FIP should be on the list of differential diagnoses.

Lesions:

Histology of lesions is usually pathognomonic and is traditionally considered the gold standard for diagnosis of FIP. H&E-stained samples typically contain localized perivascular mixed inflammation with macrophages, neutrophils, lymphocytes, and plasma cells. FCoV can be identified by immunohistochemistry in the macrophages within the lesions. Pyogranulomas may be large and consolidated, sometimes with focal tissue necrosis, or numerous and small. Lymphoid tissues in cats with FIP often show lymphoid depletion caused by apoptosis.

Rivalta’s test is a simple, inexpensive method that does not require special laboratory equipment and can be performed easily in private practice. It is very useful in cats to differentiate between effusions caused by FIP and effusions caused by other diseases. The high protein content and high concentrations of fibrin and inflammatory mediators lead to a positive reaction.

Rivalta’s test is a simple, inexpensive method that does not require special laboratory equipment and can be performed easily in private practice. It is very useful in cats to differentiate between effusions caused by FIP and effusions caused by other diseases. The high protein content and high concentrations of fibrin and inflammatory mediators lead to a positive reaction.

173
Q
A

Meningiomas

174
Q
A

D. Oral methionine (a urine acidifier)

175
Q
A

D. Tetracycline orally for 45 days.

Chlamydiosis

Severity of clinical signs and lesions depends on the virulence of the organism, infectious dose, stress factors, and susceptibility of the bird species; asymptomatic infections are common. Nasal and ocular discharge, conjunctivitis, sinusitis, green to yellow-green droppings, fever, inactivity, ruffled feathers, weakness, inappetence, and weight loss can be seen in clinically affected birds. Clinical pathology test results vary with the organs most affected and severity of the disease. Hematologic changes most often present are anemia and leukocytosis with heterophilia and monocytosis. Plasma bile acids, AST, LDH, and uric acid may be increased. A radiograph or a laparoscopy may reveal an enlarged liver and spleen and thickened airsacs.

176
Q
A

C. Herpesvirus isolation from affected oral tissues

177
Q
A

PGF2alpha

178
Q
A

B. Epizootic Bovine Abortion, foothills abortion

TYPICAL CLINICAL FINDINGS:

· Late term (6-8 months) abortions or birth of weak calves with high mortality

· In utero death and autolysis is rare

· Aborted fetuses have high serum immunoglobulin levels

· Cows are not ill and readily conceive again; they usually do not abort again

· The placenta is expelled normally and lesions are mild if present and involve connective tissue

179
Q
A

Pasteurella multocida- gram negative coccobacilli

Pseudomonas- gram negative rod

180
Q
A

Bias

181
Q
A

D. Ovulation in 96 hours

182
Q
A

Penicillin G

183
Q
A

D. Decreasing dose of EPO

184
Q
A

ammonium urate urolithiasis

PSS

185
Q
A

IBR or bovine herpes virus 1

Definition
LB.R. or bovine herpes virus- 1 infection, is a highly infectious disease caused by a virus, and characterized by rhinotracheitis, conjunctivitis, fever and a short course with a high recovery rate, encephalitis, the systemic form of the disease in new born calves, infectious pustular vulvovaginitis (l.P.V.), and abortion diseases are caused by the same virus.

Etiology

  • An alphaherpesvirus known as bovid herpesvirus-1 which is capable of growth on tissue culture and producing the clinical forms of the disease complex.
  • Antigenic differences between isolates of the virus may account for some of the diverse epidemiological and pathological patterns of behaviour of this herpes virus.

Epidemiology

1-Occurrence
Disease complex caused by this virus have been recognized in most cattle raising countries of Europe, Asia, North America, Africa, and inAustralia, and Newzeland.
- The respiratory form of the disease appears to occur most commonly in cattle confined in feedlots, dairy farms, and beef farms in which there is no routine vaccination program.
- Seroprevalence surveys indicate that 10 � 50 %, or even higher, of cattle are serologically positive to the virus depending on vaccination practices in individual herds, and the frequency of contact between infected and non infected animals.
- All ages and breeds of cattle are susceptible on experimental challenge, but the disease occurs naturally mostly in animals over 6 months of age probably because of their greater exposure. There is no seasonal variation in incidence, except possibly a higher cccurrence in feedlot cattle in fall and winter months when large numbers of susceptible animals are assembled.
- The disease can affect swine naturally in both respiratory and genital forms.
- There is evidence of herpes virus infection in wild ruminants, the infection may be endemic in white-tailed deer,
- The disease occur naturally in goats.

  • The disease is widespread in African wild life particularly buffalo, what may play an important rule in the maintenance of the infection among the wild life population.
  • Wild life may serve as reservoir for the virus.

2- Morbidity and case Fatality
The uncomplicated form of the respiratory disease in cattle is not hig1y fatal most losses being due mainly to secondary bacterial broncho-pneumonia.
- The morbidity rate in dairy cattle is 8 % and the case fatality, rate about 3 %.
In feedlot cattle the morbidity rate is usually 20 � 30 % in unvaccinated animals and may rarely reach 100 %, the case fatality is invariable due to 2nd bacterial tracheitis and bronchopneumonia and may reach 10 % but usually no more than I %.
- Morbidity and mortality are higher in feedlot cattle than in dairy herds because of the frequent introduction of susceptible animals into an enzootic situation.
- The case fatality rate in the systemic form of the infection in newborn calves is almost 100 %.

3- Methods of transmission
The main source of infection is the nasal exudate and coughed � up droplets, genital secretions, semen, fetal fluids and tissues.

4- Immune mechanisms
- lmmunity to IBR is compex and consists of relationship between local and systemic antibody and cell- mediated immunity
- Following natural infection or vaccination with the modified live virus vaccine, both cell-mediated and humoral components of the immune system are activated.
- Animals with low levels of antibody may be immune because of cell mediated immunity which can be evaluated using the delayed-type hypersensitivity test.
- Calves acquire colostral antibodies from dams with humoral antibody. The duration of the colostral immunity varies from 1 � 6 months of age and is dependent on the initial level transferred to the calf.
The presence of maternal antibody in he calf may interfere with the successful vaccination of calves before 6 months of age.

5- Economic importance

  • Losses are incurred due to epidemics of aborticn, infertility due to I.P.V and balanoposthitis in bulls, loss of production and deaths from the respiratory form of the disease in all ages of cattle, deaths from the highly fatal systemic form of the disease in newborn calves.
  • Cost of treatment when 2nd bacterial infection of the respiratory tract occur.

Pathogenesis

  • In the respiratory disease the virus multiplies in the nasal cavities and upper respiratory tract, resulting in rhinitis, laryngitis, and tracheitis.
  • Intratracheal adminstration of the virus results in almost complete denudation of tracheal columnar cells which presumbly has an adverse effect on the defence mechanisms of the respiratory tract.
  • Spread from the nasal cavities to the ocular tissues occurs by way of the lacrimal ducts and causes conjunctivitis with edema & swelling of the conjunctiva, multifocal plaque formation on the conjunctiva prepheral corneal edema and deep vascularization.
  • Spread from the nasal mucosa via the trigeminal nerve to the trigeminal ganglion may occur resulting in nonsuppurative encephalitis.
  • Systemic invasion of the virus is followed by localization of the virus in several different tissues.
  • The virus may be transported by prepheral leukocytes to the placenta and transferred to the fetus to cause abortion.
  • Infection in the last trimester of gestation may result in mummification, abortion, stillbirth, or weak calves with the usual lesions of I.B.R.
  • The systemic form of the infection in newborn calves is characterized by severe inflammation and necrosis of the respiratory and alimentary tracts including the Pharynex, esophagous, lungs, larynx, L.n, liver, and nephritis and encephalitis.
  • There s severe laryngeal edema and respiratory distress which results in difficulty in swallowing and aspiration pneumonia.
  • Severe highly fatal syndrome characterized by diffuse erosion and ulceration of the upper alimentary tract including the oral cavity.
  • In vitro studies indicate that BHV-1 virus can interfere with the function of effector cells, such as macrophages, neutrophils, and lymphocytes.
  • The virus also cause varying degrees of obstructive lung disease resulting in increased resistance to breathing, retention of carbon dioxide and increased resting lung volume, excessive airway constriction and impairment of broncheal relaxation occurs which may compromise lung defense mechanisms and allow development of secondary bacterial pneumonia.
  • The intrauterine inoculation of the BHV-1 into cattle results in an acute necrotizing endometritis in the uterine body and caudal portion of the horn
  • Experimentally dead infection during early pregnancy (7-28 day�s) will cause oophorits and in some cases embryonic mortality.

Clinical findings
Experimentally the incubation period of 3 - 7 days, but it may reach 10 - 20 days in feed lot cattle.
- There is considerable variation in the severity of clinical signs following natural infection with the l.B.R virus. This depend on the strain of the virus, age susceptibility, and environmental factors.
- There is sudden onset of severe signs including, anorexia, fever (up to 42 �C), severe hyperemia of the nasal mucosa with numerous clusters of grayish foci of necrosis on the mucous, membrane of the nasal septum, serous discharge from the eyes & nose, increased salivation and a degree of a hyperexitability.
- Drastic fall in milk yield in dairy herds.
- Respiration increased in rate but shallow
- Short explosive cough has been charactenistc of some outbreaks but is not recorded in others.
- Sudden death within 24 h. after 1st signs appears as a results of obstructive bronchiolitis.
- In dairy cattle in which the disease assumes its mildest form, signs may not increase beyond this stage, the temperature returning to normal in day or two and recovery being complete in 10 - 14 days.
- In feedlot cattle the illness is more loner, the febrile period is longer, the nasal dischrge become profuse and purulent and the convalesence period is longer. Some deaths may occur in the acute febrile period but most fatalities are due to a secondary brnchopneumonia.
- Some recovered cows have a persistent snoring respiration and a grossly thickned rough nasal mucosa accompanied by nasal discharge.
Conjunctivitis s a common but not constant sign, it may affect one or both eyes, the lesion is confined to conjunctiva and no invasion of the cornea.
the conjunctiva red and swollen, rrofuse occular discharge, there may be corneal odema, but no corneal ulceration, this may persist for few days.
- Calves less than 6 months may develop encephalitis which is marked by incordination, excitment alterating with depression and high mortality rate, salivation, bellowing, convulsc,ris and blindness are also recocded.
In newborn calves, within the lst few weeks of life, the systemic form ol the disease is sever & highly fatal.
The chnical findgs include sudden anorexia, fever, excessive salivation, unilateral or bilateral conjunctivitis and rhinitis
The oral mucosa usualiy hyperimic, erosion of the soft pa!ate covered by tenacious mucous. Acute pharyngitis covered with tenacious ucopurulent exudate is characteristic.
Larynx edematous, respiratory distress, bronchopneumonia and loud breath, crackles and wheezes and pulmonary edema.
- Some affected calves will have diarrhea and moderate dehydration which has been reffered to as the alimentary form of BHV � 1 infection.
The cause of diarrhea may be due to ruminal lesions.
- Abortion is a common sequel and occurs some weeks after the clinical illness or vaccination of non-immune pregnant cows with tha modified live-virus vaccine of bovine tissue culture origin. Retention of placenta often follows. Endometrits, pcor conception can occur after insemination with infected semen.

Clinical pathoy
The virus can be detected in nasal swabs by the use of the ELISA, direct and indirect immuonofluroscnce technique, immunoperoxidase, and by electron microscopic examination which may reveal herpe-like vral particles.
- A dot-blot hybridization assay for the detection of the virus in semen is superior to cell culture techniqtes.
- Several serological tests are available for the detection of antibody and a rise in titer btween the acute and convalescentce phase of infection.
- The virus neutralization test has been widley used.
- Specific antibody against BHV-1 may be detectable in fetal fluids and increases the rate of abortion.
A definitife diagnosis can be made only by combinng clinical findings with serological results an the isolation of the virus.

186
Q
A

BVDV

187
Q
A

Sensitivity- a/a+c

% true positives by my test

5/10

188
Q
A

Maintenance fluid requirements (40 mL/kg/day for larger animals and 60 mL/kg/day for smaller animals) are added to the rehydration rate

189
Q
A

** Pansystolic and loudest over the right AV area

Shunting of blood from the left ventricle into the right ventricle or right ventricular outflow tract occurs in most animals because of the pressure gradient between the two ventricles. The magnitude of the shunt depends on the size of the defect, the ratio of pulmonary to systemic vascular resistance, and the relative compliance of the two ventricles.

Clinical findings depend on the severity of the defect and the shunt direction. A small, restrictive defect usually causes minimal or no signs. Larger defects may result in left-side CHF. Cattle are prone to developing signs of right-side failure. The development of Eisenmenger syndrome is indicated by cyanosis, fatigue, and exercise intolerance. Most animals with a restrictive ventricular septal defect have a loud holosystolic murmur heard best over the right thorax that may be accompanied by a palpable thrill. This murmur is often absent or faint when a very large defect is present or when shunting is right to left. On occasion, aortic valvular insufficiency develops secondarily, because the defect may disrupt aortic valve apposition. In these cases, a concurrent diastolic murmur may be present, and the combination systolic/diastolic murmur (“to-and-fro” murmur) may be mistaken as that of a PDA. Thoracic radiographs can demonstrate generalized cardiomegaly with overcirculation of the pulmonary vessels. The defect can usually be visualized by echocardiography, although small defects may be missed. Doppler echocardiography can often confirm the presence of a shunt.

190
Q
A

A. Dorsal midline between the neck and sacrum

191
Q
A

A. Common grooming

192
Q
A

C.

193
Q
A

Nontonsillar squamous cell carcinomas are locally invasive with a low rate of metastasis, and the prognosis in dogs is good with aggressive and complete surgical resection, radiation therapy, or both. Tonsillar squamous cell carcinomas are aggressive and have a poor prognosis. Fibrosarcomas have a guarded prognosis because of their locally aggressive nature. Recurrence of tumor growth after resection is common.

In cats, squamous cell carcinoma has a poor prognosis unless the entire tumor can be removed, and longterm survival is usually seen only if diagnosed and treated early.

194
Q
A

Acute aflatoxicosis

Disease history, laboratory data, necropsy findings, and microscopic examination of the liver should indicate the nature of the hepatotoxin, but hepatic changes are somewhat similar in Senecio poisoning (see Pyrrolizidine Alkaloidosis). The presence and levels of aflatoxins in the feed should be determined. Acutely affected animals have increases in liver enzymes (alkaline phosphatase, AST, or ALT), bilirubin, serum bile acids, and prothrombin time. Chronic exposure can cause hypoproteinemia (including decrease in both albumin and globulin). Aflatoxin M1 (principal metabolite of aflatoxin B1) can be detected in urine, liver, kidney, or milk of lactating animals if toxin intakes are high. Aflatoxin residues in organs and dairy products generally are eliminated within 1–3 wk after exposure ends.

**ne effective binder for aflatoxins is hydrated sodium calcium aluminosilicates (HSCAS), which reduce the effects of aflatoxin when fed to pigs or poultry at 10 lb/ton (5 kg/tonne).

195
Q
A

C. Taenia

196
Q
A

EVA- fever, depression, anorexia, leukopenia, dependent edema (lower hind limbs, extremities, scrotum, prepuce in the stallion), conjunctivitis, supra- or periorbital edema, nasal discharge, respiratory distress, skin rash, temporary subfertility in affected stallions, abortion and death in young foals. Some colts and stallions become carriers and semen shedders after infection with EAV.

* RNA virus- arterivirus

* Respiratory exposure, venereal, congenital or indirect means (most resp dissemination in the acute phase)– acutely infected mare or chronically infected stallion

* gross findings: edema, congestion, hemorrhages esp. in subcutis of limbs and abdomen, excess peritoneal, pleural and pericardial fluid… edema and hemorrhage of the intra-abdominal and thoracic lymph nodes and the SI and LI

…. pulm edema, emphysema, enteritis, infarcts in the spleen

Partly autolyzed aborted fetuses

** vasculitis, smaller arterioles and venules

* Diagnosis: Immunohistochemical examination for viral antigen or antibody response from sera collected 3-4 weeks apart

197
Q
A

Left paralumbar– including rumenotomy

198
Q
A

a/a+c… 105/110 = 95%

199
Q
A

TMS can cause loss of skin… discontinue to be safe!

200
Q
A

Peripheral neuropathy- Facial nerve (CN 7)

201
Q

Where are enteroliths usually found?

A

Right dorsal and transverse colon, sometimes the small colon

202
Q
A

Chronic renal failure

203
Q
A

If a cow is to calve consistently, she must deliver her first calf early. Puberty is a function of breed, age, and weight. Beef heifers that are bred at 13–15 mo and calve at 22–24 mo have two advantages: they get closer attention from herdspeople by calving before the main herd starts to calve, and subsequently they have the extra time needed to rebreed with the mature cow herd. For heifers to breed at 14 mo, they should have attained at least 65%–75% of their projected mature weight;

Angus will be about 108 kg mature weight therefore they are only about 50%

** old enough but too small

204
Q
A

D. Submit a FIV western blot to confirm

205
Q
A

D or E– because Metronidazole and Orbifloxacin are concentration dependent with prolonged persistent effects.. but probably E– fluoroquinolone.

206
Q
A

D. Mammary adenocarcinoma

207
Q
A

** Colitis– Febendazole– Psyllium– Novel Proteins

Diagnosis:

The initial approach should include a complete history and physical examination, including rectal palpation and evaluation of feces. Fecal smears for Giardia and fungal elements (Histoplasma capsulatum, Pythium insidiosum), fecal flotation for parasite identification (Trichuris vulpis in dogs, Tritrichomonas foetus in cats), and culture for bacteria (Campylobacter, Salmonella, Clostridium) are suggested in cases of chronic colitis. Rectal cytology is an important tool to exclude other causes of large-bowel diarrhea. It can reveal inflammatory cells, neoplastic cells, and certain infectious agents (eg, H capsulatum). Cases of suspected clostridial colitis (>5 endospores per field) should be confirmed by identifying Clostridium perfringens enterotoxin A and B in feces using a commercially available ELISA after a fecal bacterial culture is performed.

A dietary trial is recommended before pursuing more advanced diagnostics. If clinical signs persist, a CBC, biochemical profile, and urinalysis should be performed to exclude other diseases; however, in most cases of chronic colitis, the results are normal. For cats, feline leukemia virus/feline immunodeficiency virus testing is also recommended as well as a thyroid level if age appropriate. Routine abdominal radiographs are also usually normal. Contrast radiographs may occasionally demonstrate intraluminal narrowing, which could indicate an infiltrative disease process. Ultrasonography allows the visualization of colonic mucosa, localized lesions, and the size and echogenicity of lymph nodes.

208
Q
A

* Klebsiella

Control
• Frequent manure removal
• Using appropriate free stalls and managing the bedding
• Providing pastured cows with clean locations to lie down
o Sand is a preferred bedding material
o Wood products are to be avoided as they predispose to Klebsiella mastitis
• Pre-dipping teats and having clean dry udders at milking are important

209
Q
A

D. Number of open cows beyond the voluntary waiting period and number of breedings

210
Q
A

d/d+b

579/900

211
Q
A

$133.00, would actually be $100

212
Q

What is the definition of FPT in a foal?

A

Immunologic support in the form of IV plasma transfusion (1–2 L) is also indicated to raise the IgG level to >800 mg/dL.

213
Q
A

** Inhalation

Canine distemper is caused by a paramyxovirus closely related to the viruses of measles and rinderpest. The fragile, enveloped, single-strand RNA virus is sensitive to lipid solvents, such as ether, and most disinfectants, including phenols and quaternary ammonium compounds. It is relatively unstable outside the host. The main route of infection is via aerosol droplet secretions from infected animals. Some infected dogs may shed virus for several months.

214
Q
A

ynchronous diaphragmatic flutter is due to firing of the phrenic nerve in synchrony with atrial depolarization, causing the diaphragm to contract with each heartbeat. This occasionally produces an audible thumping sound. Inciting causes include endurance exercise, hypocalcemia, hypoparathyroidism, digestive disturbances, and repeated administration of calcium-containing fluids to performance horses. Synchronous diaphragmatic flutter may be a singular occurrence or a chronic recurring problem. The most consistently reported metabolic derangement is low serum ionized calcium concentrations usually associated with hypochloremic metabolic alkalosis. Metabolic alkalosis may alter the ratio of free to bound calcium (increasing calcium binding to protein and decreasing ionized calcium), which possibly induces diaphragmatic flutter.

Most horses undergo rapid remission of signs when given calcium solutions IV. Although hypomagnesemia is often present with synchronous diaphragmatic flutter, horses do not respond to magnesium supplementation unless calcium is administered concurrently. Response to therapy is also reflected by improved mental status, return of appetite, and gut motility. For horses with chronic diaphragmatic flutter, providing chloride, potassium, sodium, calcium, and magnesium during prolonged exercise may help reduce fluid losses and the metabolic alkalosis. Alternative approaches involve reducing dietary calcium for a few days before competition in horses prone to diaphragmatic flutter. This reduction in dietary calcium may stimulate the endocrine homeostatic mechanisms and increase osteoclastic activity. Limiting alfalfa hay, which has a relatively high calcium concentration, may be indicated in chronically affected horses.

215
Q
A

Chronic Laminitis

216
Q
A

C. IV fluids

217
Q
A

Levator palpebral muscle is innervated by Oculomotor (III)

218
Q
A

In cattle and horses, accidental perforation during rectal examination necessitates immediate treatment to reduce the risk of peritonitis and death. Exploration throughout the abdomen should be slow, deliberate, and smooth. The temptation to use the fingertips excessively or to push the arm through a region of resistance must be avoided. Rectal tears in horses have been classified according to the tissue layers penetrated. Grade I tears involve the mucosa or submucosa. Grade II tears involve rupture of the muscular layers only. Grade III tears involve mucosa, submucosa, and muscular layers, including tears that extend into the mesorectum. Grade IV tears involve perforation of all layers of the rectum and extension into the peritoneal cavity.

Grade I tears may be treated conservatively with broad-spectrum antibiotics and IV fluids. Flunixin meglumine may be given to prevent or treat endotoxic shock. Mineral oil is given via stomach tube to soften feces, and the diet should consist of pasture grasses or alfalfa. Grade II and III tears require immediate and more extensive surgery. A consultation with a specialist is suggested immediately after diagnosis. Grade IV tears carry a grave prognosis; they should be repaired only if small and if treatment is instituted before the peritoneal cavity is grossly contaminated.

219
Q
A

Potassium

inorganic phosphorus concentrations are increased. Potassium depletion, due to renal potassium wasting combined with inadequate intake and the kaliuretic effects of acidosis, is frequently seen in cats and occasionally in dogs. Hyperkalemia associated with oliguria and anuria may be noted in terminal Stage 4 or whenever marked prerenal azotemia is concurrent with CKD. Systemic hypertension and associated complications develop in ~20% of affected cats and dogs and can occur at any stage. Osteoporosis may be seen radiographically, although this late finding is generally not helpful for diagnosis.

220
Q
A

Mastitis

221
Q
A

Cutaneous habronemiasis is a skin disease of Equidae caused in part by the larvae of the spirurid stomach worms (see Gastrointestinal Parasites of Horses). When the larvae emerge from flies feeding on preexisting wounds or on moisture of the genitalia or eyes, they migrate into and irritate the tissue, which causes a granulomatous reaction. The lesion becomes chronic, and healing is protracted. Diagnosis is based on finding nonhealing, reddish brown, greasy skin granulomas that contain yellow, calcified material the size of rice grains. Larvae, recognized by spiny knobs on their tails, can sometimes be demonstrated in scrapings of the lesions. Many different treatments have been tried, most with poor results. Symptomatic treatment, including use of insect repellents, may be of benefit, and organophosphates applied topically to the abraded surface may kill the larvae. Surgical removal or cauterization of the excessive granulation tissue may be necessary. Treatment with ivermectin (200 mcg/kg) has been effective, and although there may be temporary exacerbation of the lesions (presumably in reaction to the dying larvae), spontaneous healing may be expected. Moxidectin at 400 mcg/kg also appears to be active against Habronema spp in the stomach. Control of the fly hosts and regular collection and stacking of manure, together with anthelmintic therapy, may reduce the incidence.

222
Q
A

Glasser’s disease (H parasuis)

At necropsy, polyarthritis and polyserositis are seen with both mycoplasma infection and Glässer’s disease, and pneumonia may have developed. The initial, exudative response is usually serous or serofibrinous with a mycoplasmal infection; however, it is fibrinous or fibrinopurulent with Glässer’s disease. Hence, M hyorhinis causes a mild synovitis with villous hypertrophy and hyperplasia; an excess of clear, yellow, or brown synovia; and a serofibrinous pericarditis, pleuritis, and peritonitis. Otitis media has also been reported. With H parasuis, a fibrinopurulent synovitis with periarticular edema, polyserositis with pseudomembranes, and sometimes fibrinopurulent meningitis are seen. The articular surfaces are usually unaffected in either condition.

223
Q
A

D. 41-50 (42-45)

224
Q
A

Anaphylaxis or Type I hypersensitivity reactions in cattle can result in an atypical interstitial pneumonia. The lung is a major target organ in cattle for Type I hypersensitivity. Clinical signs are those of acute respiratory distress. Cattle that die of anaphylaxis may have lesions consistent with those described for atypical interstitial pneumonia. Treatment is administration of epinephrine; supportive treatment includes anti-inflammatory therapy with corticosteroids or NSAIDs. If pharyngeal or laryngeal edema is present, a tracheostomy may be indicated.

225
Q
A

Colibacillosis, clostridial enterotoxemia, coccidiosis, TGE, rotavirus

A or B

226
Q
A

Histophilosis, or Histophilus somni–associated disease, is a common disease in North American cattle. It also has been reported to occur sporadically in beef and dairy cattle worldwide. H somni predominantly causes an acute, often fatal, septicemic disease that can involve the respiratory, cardiovascular, musculoskeletal, or nervous systems, either singly or together in confined cattle. The reproductive system is often affected without clinical signs or other systemic involvement; however, herd infertility has been reported to occur more frequently.

Feedlot cattle that die of possible histophilosis should be examined at necropsy. These animals may exhibit an array of postmortem findings, including fibrinous pleuritis without bronchopneumonia, a focal myocardial lesion (often in the papillary muscle of the left ventricle), fibrinous pericarditis, bronchopneumonia, polyarthritis, and a fibrinous laryngitis. Less common gross postmortem lesions include polyserositis, fibrinous gonitis, and a fibrinopurulent meningitis. Probably the most common lesion seen in the feedyard is that of “left heart failure” associated with a marked necrotic sequestrum in the myocardial wall.

In animals that survive long enough to allow the pathology to progress, the fibrinous portion of the lesions becomes fibrotic, and the infarctions or sequestrae in the heart or larynx liquefy and wall off to become an abscess. Lesions of the reproductive tract may include suppurative vaginitis, cervicitis, and endometritis.

A definitive diagnosis is based on sampling and examination of affected tissues collected during a necropsy or clinical examination. A diagnosis at necropsy of “left heart failure” with a concomitant myocardial lesion or a carcass in which the thoracic space is filled with fluid and fibrin with little pneumonia is considered definitive for histophilosis. Historically, isolation of the organism from CSF, brain, blood, urine, joint fluid, or other sterile, internal organs or fluids has been used to confirm the diagnosis. Because H somni is a commensal of the mucous membranes of cattle, the bacterium should be isolated in predominant or pure culture from the respiratory or urogenital tract to be considered a significant etiologic agent. This may be difficult, because antimicrobial treatment often interferes with recovery of the organism. The characteristic histologic lesion is suppurative with heavy infiltrations of neutrophils in all tissues where localization of the bacteria occurs. Currently, the diagnosis is usually confirmed with molecular techniques such as immunohistochemical staining of H&E-stained tissues or a fresh lesion swab subjected to a specific PCR test.

major hindrance to successful treatment of individual histophilosis cases is the difficulty in identifying affected animals early in the course of disease because of its often rapidly fatal nature. Antimicrobial treatment is most effective in the early stages of disease. Florfenicol (20 mg/kg, IM, repeated in 48 hr, or 40 mg/kg, SC, once) may be the antimicrobial of choice if histophilosis is the tentative diagnosis in an individual animal.

227
Q
A

A number of herpesviruses affect nonhuman primates; many exist as latent or subclinical infections in reservoir hosts but cause severe disease or death when transmitted naturally to other hosts. All macaques are considered to be potential shedders of Cercopithecine herpesvirus type 1 (Herpesvirus simiae, B virus). The infection is generally subclinical or mild (conjunctivitis or oral vesicles) in Macaca spp but usually causes a fatal encephalitis and encephalomyelitis in people. Transmission may occur via a bite, scratch, or contamination of a superficial wound or mucous membranes (eg, conjunctiva) with infectious saliva, conjunctival secretion, or genitourinary secretions. Human fatalities due to B virus encephalitis illustrate the importance of using appropriate precautions and personal protective equipment to prevent direct or indirect contact with macaque secretions and body fluids. Captive-born and raised animals should still be screened when first presented in a clinical setting and subsequently every year afterward in a private facility.

228
Q
A

In animals with detrusor hyporeflexia or bladder atony, bethanechol chloride may be of some benefit. This cholinergic agonist stimulates the initiation of detrusor muscle contraction. The dosage for dogs is 5–25 mg/dog, PO, tid, and for cats is 2.5–7.5 mg/cat, PO, tid.

229
Q
A

Baylisascaris procyonis

, an intestinal nematode of raccoons, can cause severe

neurological and ocular signs when its larvae migrate in humans, other mammals and

birds. Although clinical cases seem to be rare

in people, most reported cases have

been serious and difficult to treat. Severe disease has also been reported in other

mammals and birds. Other spec

ies of

Baylisascaris

, particularly

B. melis

of European

badgers and

B. columnaris

of skunks, can also caus

e neural and ocular larva migrans

in animals, and are potential human pathogens.

230
Q
A

Lungs

231
Q
A

A particularly malignant form of cancer, carcinoma is characterized by its ability to spread quickly throughout the body. Adenocarcinoma is differentiated only in that it originates in the glandular tissue. Adenocarcinoma of the thyroid gland is a malignant tumor, which can metastasize to other tissue and organs, including the lungs.

232
Q
A

* B. Palpation of the chorionic vesicle

At 30 days, the uterine horns are small with pronounced tone, and the conceptus can be felt as a ventral bulge 4 cm in diameter and positioned at the base of the gravid uterine horn. The uterine wall is thin over the expanding conceptus.

233
Q
A

* Bronchoscopy (or bronchiol wash)

The diagnosis is made from the history and clinical signs and by elimination of other causes of coughing. In chronic bronchitis, chest radiographs may show an increase in linear and peribronchial markings. Bronchoscopy reveals inflamed epithelium and often mucopurulent mucus in the bronchi. In addition, the procedure allows collection of biopsy and swab samples for in vitro assay. Bronchial washing is an additional diagnostic aid that may demonstrate causative agents or significant cellular responses (eg, eosinophils).

234
Q
A

Leptospira

235
Q
A

Luteal cystic ovary disease is characterized by enlarged ovaries with one or more cysts, the walls of which are thicker than those of follicular cysts because of a lining of luteal tissue. Incidence ratios of follicular versus luteal cysts vary greatly because of diagnostic tendencies of individual veterinarians. Classically, luteal COD is defined as the presence of a fluid-filled ovarian structure >25 mm diameter persisting >7 days in the absence of a CL and with a wall diameter >3 mm, usually associated with abnormal reproductive signs. Normal lacunae formation in CL may be incorrectly classified as luteal COD.

The treatment of choice is luteolytic doses of PGF2α if a correct diagnosis can be ascertained. A normal estrus is expected in 3–5 days. The major limitation of this treatment is the difficulty in accurately estimating the amount of luteal tissue present. If the structure being diagnosed as a luteal cyst is really a developing CL (as discussed above, sometimes called a cystic CL), it may not respond because dairy cows do not become highly responsive to the luteolytic action of PGF2α until day 6 after estrus. Ultrasound examination is increasingly common and facilitates diagnosis of ovarian structures. Luteal cysts also respond to human chorionic gonadotropin and GnRH therapy that is effective in the treatment of follicular cysts, but the next estrus could occur 5–21 days after treatment. Manual rupture of luteal cysts is not recommended because of the risk of trauma and hemorrhage. Because of poor estrus detection practices on many dairy farms, the treatment of choice for both follicular and luteal cysts is intravaginal progesterone/prostaglandin (a fixed timed artificial insemination protocol) (see Treatment of Follicular Cystic Ovary Disease in Large Animals). Application of this protocol in affected cows promotes timely breeding after treatment.

236
Q
A

α-mannosidosis - Aberdeen Angus, Murray Grey, Brangus, Simmental, Galloway, and Holstein cattle[3].

β-Mannosidosis - primarily Saler cattle[4].

237
Q
A

B. Calcium oxalate and magnesium ammonium phosphate (struvite)

238
Q
A

Slab fracture of the right third carpal bone

239
Q
A

Diagnosis is by demonstration of the characteristic eggs in the feces, but because the discharge of proglottids is sporadic, a single fecal examination may not be diagnostic. In light infections, no signs of disease are present; in heavy infections, GI disturbances may be seen. Unthriftiness and anemia have been reported. Ulceration of the mucosa is quite common in the area of attachment of A perfoliata and has been suggested as one cause of intussusception. Intestinal perforation, peritonitis, and subsequent colic have been associated with Anoplocephala infections. Colic from disturbances of the ileocecal area is more likely in horses with tapeworm infections than in those not infected. Colic associated with tapeworm infections often recurs. The site of attachment of tapeworms frequently becomes secondarily infected or abscessed. A

** pRAZIQUANTEL OR PYRANTEL

240
Q
A

The bacterial agents most commonly linked with BRD are Mannheimia haemolytica, Pasteurella multocida, Histophilus somni, and Mycoplasma bovis.[5] Viral agents include Bovine Viral Diarrhea (BVD), Infectious Bovine Rhinotracheitis (IBR), Bovine Respiratory Synctial Virus (BRSV), and Parainfluenza Type-3 Virus (PI-3).[7]

241
Q
A
242
Q
A

Pancytopenia- v+d are more common

243
Q
A

The distal aspect of the sigmoid flexure of cattle and the sigmoid flexure and urethral process of sheep and goats are the most common sites

244
Q
A

Generally, this finding is considered normal and is not a cause for concern unless accompanied by straining to urinate, slow urination, blood in the urine, depression, poor appetite or other abnormal behavior.

Rarely, urine sediments can turn into a heavy sludge that is never completely voided, and this can be a problem.

245
Q
A

Salivary mucocele is an accumulation of saliva outside of the salivary gland that is due to rupture of a salivary duct.

A ranula is a collection of saliva beneath the tongue and is due to rupture of the sublingual salivary duct.

246
Q
A

HYPP- inherited autosomal dominant that affects sodium channels in muscle cells and the ability to regulate potassium levels in the blood. I

Some horses are more affected by the disease than others and some attacks will be more severe than others, even in the same horse. Symptoms of an HYPP attack may include:

Muscle trembling

Prolapse of the third eyelid — this means that the third eyelid flickers across the eye or covers more of the eye than normal

Generalized weakness

Weakness in the hind end — the horse may look as though it is ‘dog-sitting’

Complete collapse

Abnormal whinny — because the muscles of the voicebox are affected as well as other muscles

Death — in a severe attack the diaphragm is paralyzed and the horse can suffocate

HYPP attacks occur randomly and can strike a horse standing calmly in a stable just as easily as during exercise. Following an HYPP attack, the horse appears normal and is not in any pain which helps to distinguish it from Equine Exertional Rhabdomyolysis (ER), commonly known as “Azoturia,” “Monday Morning Sickness” or “tying up.”

247
Q
A

Diagnosis is often made from the signalment, history, clinical signs, and response to treatment. A pretreatment total serum calcium concentration <7 mg/dL (<6 mg/dL in cats) confirms the diagnosis.

Slow IV administration of 10% calcium gluconate is given to effect (0.5–1.5 mL/kg over 10–30 min; 5–20 mL is the usual dose). This usually results in rapid clinical improvement within 15 min. Muscle relaxation should be immediate.

248
Q
A

A. Or pushed into the stomach if cannot remove with forceps

249
Q
A

Epistaxis

250
Q
A

** Peritonitis

FIP

Usually, the protein content is very high (>3.5 g/dL), consistent with an exudate, whereas the cellular content is low (<5,000 nucleated cells/mL), resembling a modified transudate or even pure transudate. Major differential diagnoses for these effusions include inflammatory liver disease, lymphoma, heart failure, and bacterial peritonitis or pleuritis. LDH activity typically is high (>300 IU/L). Cytology is variable but often consists predominantly of macrophages and nondegenerate neutrophils (in much lower numbers than seen with bacterial infection). These effusions can usually be differentiated from bacterial infection or lymphoma by the presence of malignant cells, degenerate neutrophils, or intracellular bacteria on cytology and bacterial growth on culture, respectively. The albumin to globulin ratio of the effusion can be measured: a ratio of <0.5 is strongly correlated with FIP, with a PPV between 66% and 95%, depending on the prevalence of FIP in the cat’s environment. An albumin to globulin ratio >0.81 has a 100% NPV, essentially excluding FIP.

251
Q
A

Penicillin IM

S suis is a significant pathogen of swine and one of the most important causes of bacterial mortality in piglets after weaning. It is considered a normal inhabitant of the upper respiratory tract (especially nonvirulent strains) and can be easily found in tonsils, which are considered a natural niche. It can also be isolated from the reproductive and GI tracts of clinically healthy pigs.

252
Q
A

Legume dominated pasture

253
Q
A

D. Internal PTH gland

254
Q
A

Cerebellar signs ipsilateral

255
Q
A

FIP

Clinical signs associated with the non-effusive form depend on the body system affected and are due to localized perivascular infiltrates of inflammatory cells (pyogranulomas) in the parenchyma of organs. Thoracic or abdominal effusions are either absent or too scant to be appreciated clinically. The most common clinical presentations involve the eyes or CNS. Ocular involvement may manifest as anterior uveitis with hyphema, hypopyon, aqueous flare, miosis, and keratic precipitates. Other findings include an irregularly shaped pupil, a change in iris color, chorioretinitis, retinal hemorrhage, or retinal detachment(6). Ocular disease may be the sole manifestation of FIP in affected cats, or it may be combined with CNS or abdominal involvement.

256
Q
A

** E. Restriction of pasture grazing and corticosteroid therapy

Atypical interstitial pneumonia (AIP) (‘rye grass staggers’, ‘fog fever’[1]) is a multifaceted disease with several known causes or clinical presentations. Cases of AIP are often complicated with bacterial, viral, or mycoplasmal organisms[2].

Causes are attributed to cattle grazing on lush forages high in the amino acid tryptophan, resulting in toxic lung damage and pneumonia[3]. The disorder occurs when cattle are abruptly moved from sparse dry summer pastures to lush irrigated pastures in the fall.

AIP commonly occurs as a herd outbreak with as many as 50% of cattle showing acute onset of respiratory distress. Outbreaks typically occur during the first 2 weeks of a pasture change, generally during the first few days.

Clinical signs

Clinical signs include head tremors, stiff gait, and staggering[4]. As the disease worsens, rapid, labored respiration is observed with head extended and held low. Adults are more susceptible than yearlings. Death occurs at any time during the course of clinical illness. Bacterial pneumonias consist of bronchopneumonia, fibrinous pneumonia, and pleuropneumonia as well as caseonecrotic, aspiration, and tuberculous pneumonias. Two major patterns of interstitial pneumonia are recognized in cattle, and verminous pneumonia is associated with Dictyocaulus viviparus infection[5].

During the acute phase, lesions consists of alveolar septal thickening and a massive outpouring of fibrin-rich proteinaceous fluid into alveolar spaces with formation of hyaline alveolar membranes mixed with few leukocytes (white blood cells). At about three days, proliferation of the epithelial cells (pneumocyte proliferation) lining alveoli commences and at this stage, there may be a mix of hyalin membranes and pneumocyte proliferation. As the lesions progress, pneumocyte proliferation dominates. During these stages, gross lesions include failure of the lungs to collapse when the chest cavity is opened; the lungs have a meaty texture, and edema and/or emphysema (gas bubbles) distend interlobular septae.

Treatment

Broad-spectrum antimicrobials such as tetracyclines and ceftiofur are important to minimise secondary bacterial infections.

257
Q
A

Horn flies- The common name of Haematobia irritans comes from the fact that these flies often cluster in the hundreds around the base of the horns of cattle. Spend their entire life on cattle

* Lice- also all life stages on the host

** B. the insects seldom leave the host

258
Q
A

** Night blindness, retinal vascular attenuation, tapetal hyperreflectivity, optic disk pallor

Canine PRA

diagnosis

behavior, maze test, ophthalmoscopy, ERG

no treatment exists at present

gene mapping studies

future blood screening

Night vision goggles might help!!

259
Q
A

Alkalinizing diet

260
Q
A

Bluetongue

Bluetongue virus is the type-species of the genus Orbivirus in the family Reoviridae

The course of the disease in sheep can vary from peracute to chronic, with a mortality rate of 2%–90%. Peracute cases die within 7–9 days of infection, mostly as a result of severe pulmonary edema leading to dyspnea, frothing from the nostrils, and death by asphyxiation. In chronic cases, sheep may die 3–5 wk after infection, mainly as a result of bacterial complications, especially pasteurellosis, and exhaustion. Mild cases usually recover rapidly and completely. The major production losses include deaths, unthriftiness during prolonged convalescence, wool breaks, and reproductive losses.

In sheep, BTV causes vascular endothelial damage, resulting in changes to capillary permeability and subsequent intravascular coagulation. This results in edema, congestion, hemorrhage, inflammation, and necrosis. The clinical signs in sheep are typical. After an incubation period of 4–6 days, a fever of 105°–107.5°F (40.5°–42°C) develops. The animals are listless and reluctant to move. Clinical signs in young lambs are more apparent, and the mortality rate can be high (up to 30%). Approximately 2 days after onset of fever, additional clinical signs may be seen, such as edema of lips, nose, face, submandibular area, eyelids, and sometimes ears; congestion of mouth, nose, nasal cavities, conjunctiva, and coronary bands; and lameness and depression. A serous nasal discharge is common, later becoming mucopurulent. The congestion of nose and nasal cavities produces a “sore muzzle” effect, the term used to describe the disease in sheep in the USA.

261
Q
A

Because of the degenerate neutrophils suspect bacteria– pulmonary granuloma….. BUT atypical cells.. I would say always neoplasia…

C… Pulm. adenocarcinoma

When atypical epithelial cells are observed in a urine sediment wet-mount prepara

-

tion, it is helpful in many cases to prepare a dry-mount urine cytologic specimen for

further evaluation, as thorough assessment of cell morphology is often not possible

using low magnification and a wet-mount preparation. A dry-mount urine sediment

cytologic specimen can also be easily shipped to a commercial laboratory for evalua

-

tion by a pathologist as needed.

262
Q
A

Naturally infected cats are primarily asymptomatic, subclinical carriers of Bartonella henselae. B. henselae infections in cats, also known as feline bartonellosis, may occasionally cause a self-limiting, transient, febrile illness that lasts for approximately 48-72 hours. Clinical symptoms of more serious infection, although rare, include fever, vomiting, lethargy, red eyes, swollen lymph nodes, and/or decreased appetite. Bacteremia can persist for months with clinical signs appearing when the cat is under stress (surgery or trauma) or concurrent with another disease.

Bartonella infections in symptomatic pets should be confirmed by culturing the organism from blood or tissues such as lymph node or heart valve (in cases of endocarditis) or by amplifying Bartonella-specific DNA sequences from tissues using PCR. Serology using immunofluorescence antibodies (IFA) is the most sensitive diagnostic tool for diagnosing Bartonella exposure, but it is not useful as a means of predicting animals that may pose a public health risk to their owners, or identifying animals that require treatment.

Treatment

Doxycycline, amoxicillin, enrofloxacin, and rifampin given for a long duration (4-6 weeks) may be effective in reducing the level of bacteremia in the infected cat or dog. The ability of any antibiotic or antibiotic combination to completely clear B. henselae from the blood stream has not been established. Given the duration of treatment necessary to clear the bacteremia and the concern for development of antimicrobial resistance, treatment is recommended only for pets that have clinical symptoms.

263
Q
A

Sodium hydroxide

264
Q
A

A. 0-5%

265
Q
A

C. Humans (or cattle for other kinds)

266
Q
A

Ollanus tricuspis

Ollulanus tricuspis is a small worm, ≤1 mm long, that infects several animal species, typically cats and other felids, and occasionally induces a mild erosive or catarrhal gastritis. Vomiting minutes to a few hours after eating is a common sign. The female worms are viviparous, so massive infections can build up endogenously. Transmission is via vomitus. Diagnosis is by microscopic demonstration of larvae (~500 μm) or adult worms in vomitus or stomach contents. The use of a Baermann apparatus enables the separation of the worms from ingesta, after which they are easier to observe. Parasites are rarely seen in feces, because they are usually digested before being passed. Therapeutic efficacy in cats has been demonstrated with fenbendazole (20–50 mg/kg/day, PO, for 3 days) and levamisole (5 mg/kg, SC, once), although these are not approved treatments.

267
Q
A

* Transverse and sternal flexure (or left ventral colon and pelvic flexure)

The right ventral colon is divided into sacculations that help mix and retain plant fibers until they are digested. It is positioned on the ventral aspect of the abdomen, extending from the flank region to the rib cage. The ventral colon then turns toward the left, becoming the sternal flexure and then the left ventral colon. The left ventral colon, which also is large and sacculated, passes caudally to the left flank area. Near the pelvic region, the diameter of the colon decreases markedly, and the colon folds back on itself. This region, called the pelvic flexure, is the initial portion of the unsacculated left dorsal colon. Presumably because of the abrupt decrease in diameter, the junction between the left ventral colon and pelvic flexure is the most common location for impactions.